Top Banner
148
Welcome message from author
This document is posted to help you gain knowledge. Please leave a comment to let me know what you think about it! Share it to your friends and learn new things together.
Transcript
Page 1: Mitul's 2007 GMAT Sep
Page 2: Mitul's 2007 GMAT Sep

09.06

 ScoreTop:VIP Only Section:09.06

Topic: September Math (Updated on 09/27/06)

Forum Jump  

Author Message Prev Topic | Next Topic 

masterAdmin

Joined: 23 September 2004

United StatesPosts: 407Gender: Male

Posted: 30 August 2006 at 4:59pm | IP Logged

Please do NOT post comments/questions on this thread.  Our admins will delete them all.

 

Disclaimer

Those questions in VIP are collected from the internet, and we just rephrased and reformatted them in a way for easier understanding. We are not, in any way, responsible for the contents and accuracy of any questions. Because they are freely available on the internet, posted by regular users on various of forums/websites, we can not verify the sources of these questions, but from our experience, we understand, by practising those questions, you will improve your ability to tackle GMAT questions.

Please refer to the Discussion link underneath each question if one is available.

1. A 12 feet x 8 feet wall is completely covered with some square tiles. The dimension of the tile is 4*4, in inches. A figure shows that each tile has 16 small squares, including 4 blue and 12 white. What is the total number of the blue squares on the wall?

1 feet = 12 inch, I think the answer should be: (12*8*12*12/4*4)*4

Discussion Link

2. A mechanic system consists of a large gear and a little gear. The rating life of the gear is 60,000 rounds. What is the   difference between   the large gears' life   and the little gears' life? [The two gears joggled to each other?]

1). The ratio between two radius is ¡

2). It cost 2 minutes for the little gear to rotate one circle.

Page 3: Mitul's 2007 GMAT Sep

Discussion Link

A similar old question in August:

A mechanic system consists of a large gear and a little gear. The rating life of the gear is 6000,000 rounds. What is the life of the whole system?

1). The ratio between two radius is ...

2). It cost 2 minutes for the little gear to round one circle.

3. 50 students of a certain class took a test. How many of them got a score less than 82?

1). the average score is 82

2). the median is 82

Answer:

Statement 1 is insufficient.

Statement 2, the distribution of scores could be: 24 scores less than 82, then 82, 82, then 24 scores greater than 82. Or, 25 scores less than 82, including the largest score 81, and 25 scores greater than 82, including the least score 83. Insufficient.

Answer is E

Discussion Link

4. Sequence s1,s2,s3,..sn, is such that s=1/n-1/n+1 . Is the sum of first n terms greater than 9/10?

1). n>10

2). n<19

Discussion Link

5. Is x-y>0? (Or x+y>0?)

1). x-y/x>0

2). x+y/x-y>0

Discussion Link

6. [(4^300+2^600)/2]^4=?

Answer: 2^2400

Page 4: Mitul's 2007 GMAT Sep

Discussion Link

7. The average of some numbers is 43, what is the standard deviation?

1). The least number is 43

2). The greatest number is 43

Discussion Link

8. S(n)=S(n-1)^S(N-2), S1=1,S2=2,S3=2^1....S6/S5=?

Answer:

S6=2^16, S5=2^4, S6/S5=2^12

Discussion Link

9. The sides of a triangle is a, b, and c. Are the three angles all less than 90 measure degrees?

1). The areas of the semi-circles with the radius a, b, c are 4, 5, 6, respectively.

2). c<a+b<c+2

Discussion Link

10. Is x divisible by 3?

1). When x is divided by 5, the remainder is 1

2). When x is divided by 15, the remainder is 1

Answer:

1. x=5n+1, insufficient.

2. x=15n+1, insufficient.

Two statements together, x=15n+1, insufficient.

Discussion Link

11. There are two points p(a,b), and q(c,d) in a xy-plane. If a^2+b^2+c^2+d^2=87, and ac+bd=31, what the length of the segment pq?

Discussion Link

Edited by soft rock on 06 September 2006 at 3:18am

Page 5: Mitul's 2007 GMAT Sep

Back to Top  

soft rockGMAT Tutor

Joined: 20 October 2004British Virgin Islands

Posts: 1125Gender: Not Specified

Posted: 03 September 2006 at 5:30am | IP Logged

  Yes No UnsureA 500 200 100B 400 300 100

12. In a certain survey, each of 800 people expressed their attitude to A and B. They can say yes, no, or unsure. If 200 people said yes to A but not to B, how many people said yes to either A or B?

Discussion Link

13. This semester, each of the 90 students in a certain class took at least one course from A, B, and C. If 60 students took A, 40 students took B, 20 students took C, and 5 students took all the three, how many students took exactly two courses?

Discussion Link

14. X, y and z are three digits numbers, and x=y+z. Is the hundreds’ digit of x equal to the sum of hundreds’ digits of y and z?

1). the tens’ digit of x equal to the sum of tens’ digits of y and z

2). the units’ digit of x equal to the sum of units’ digits of y and z

Discussion Link

15. If a=x+1/x,b=x-1/x, 2^(a^2)/2(b^2)=?

Answer: 16

Discussion Link

16. In the first half of a certain trip, a car maintained an average speed of 40m/h; in the second half of the trip, the car maintained an average speed of 60m/h. What is the average speed for the whole trip?

Discussion Link

17. Is |x-2|+|x+2|<4?

1). x<0

2). x is within (-2, 2)

Discussion Link

Edited by master on 03 September 2006 at 6:07pm

Back to Top  

Page 6: Mitul's 2007 GMAT Sep

soft rockGMAT Tutor

Joined: 20 October 2004British Virgin Islands

Posts: 1125Gender: Not Specified

Posted: 04 September 2006 at 9:02am | IP Logged

18. The area of a triangle is 16*root3. What is the perimeter of the triangle?

1). It is an isosceles triangle

2). Two of the inside angles are 30 degrees.

Discussion Link

19. As the figure shows, A is the center of the circle, AB=AC=2, angle BAC is 120 degrees. What is the area of the triangle?

Discussion Link

20. The median of the rent of the apartments is 550. What's the average (arithmatic mean) of the rent?

1). The average rent of apartments with rent higher than the median is 890.

2). There are 370 apartments with the rent higher than the median.

Discussion Link

21. What is the area of the circle inscribed in the square with side 7?

Discussion Link

22. Two cubes each is numbered with 1 to 6 (called dice). Tossing the dices, what is the probability that the sum of the numbers on the top is greater than 9?

Discussion Link

23. The first term of a geometric sequence is 1/2, and the common ratio is 1/2. The 10th term will fall into which of the following range?

Page 7: Mitul's 2007 GMAT Sep

0.1~0.01, 0.01~0.001, 0.001~0.0001,0.0001~0.00001

Discussion Link

24. The sides of a triangle is a, b, and c. Is it a right triangle?

1). a^2+b^2=3^2+4^2

2). a^2+c^2=3^2+5^2

Discussion Link

25. -1<x<-3/4, which of the following has the greatest value?

-x, -1/x, 1-1/x, 1/(1+x)

Discussion Link

26. A list of consecutive natural numbers contains 9 and 15. Does the list contain number which can be divisible by 8?

1). The list has 10 numbers.

2). 18 is in the list.

Discussion Link

27. In the xy-plane, line A has slope a and line B has slope b. Are the two lines parallel to each other?

1). 4^a=2^b

2). 2^a=3^b

Discussion Link

28. If x and y are nonzero, is (x^3)(y^5)/(x^2)(y^3)>0

1). x>0

2). y>0

Discussion Link

29. Each segment from A to G has equal distance. Which of the following point represents [-(3^12)] ?

Page 8: Mitul's 2007 GMAT Sep

Discussion Link

Edited by GMATer on 04 September 2006 at 9:50pm

Back to Top  

soft rockGMAT Tutor

Joined: 20 October 2004British Virgin Islands

Posts: 1125Gender: Not Specified

Posted: 05 September 2006 at 10:11am | IP Logged

30. A parallelogram with area of 18 has a side of 6. Which of the following could be one of its inside angles?

30, 50, 60, 70, ...

Discussion Link

31. A ship started off at the rate of a. t hours later, in the same direction, another ship started off at the rate of b. In how many hours the second ship will catch up with the first ship?

Discussion Link

32. In the triangle ABC, angle A is 30 degrees, and AD is perpendicular to AC. What is the perimeter of the triangle?

1). given the perimeter of the triangle ABD.

2). Given the perimeter of the triangle BCD.

Discussion Link

33. What is the units' digit of (3^11)(4^13)+1?

Discussion Link

34. Some toys include large, middle, and small model with red, yellow, green, or blue color. If numbers of all model-color combinations are the same, for example, number of red large toys is equal to number of green little toys. A boy wants a red-large toy. If his mother select one for him at random, what is the probability that at least one of the color and model

Page 9: Mitul's 2007 GMAT Sep

will satisfy the boy?

Discussion Link

Discussion Link2

35. Is point C has the same distance from point A(-3,-3) and point B(1,-3)?

1). C lies on the line x=-1

2). C lies on the line y=-3

Discussion Link

36. One of the solutions of x^2-5x+t=0 is -1. What is the other solution?

Discussion Link

Edited by GMATer on 07 September 2006 at 10:51pm

Back to Top  

soft rockGMAT Tutor

Joined: 20 October 2004British Virgin Islands

Posts: 1125Gender: Not Specified

Posted: 05 September 2006 at 10:52pm | IP Logged

37. If 3^a*4^b =576, where a and b are integers, 4^a*3^b=?

Discussion Link

38. The area of a triangle with the sides x and y is yx/2. What is the angle formed by x and y?

Discussion Link

39. People A and B together can finish 800 products in x hours. If it takes y hours for A alone to finish the 800 products, how many hours will it take for B alone to finish the 800 products?

Discussion Link

40. Tickets for a movie include A, with rate of $4.5, and B, with rate of $6. If 60% of the tickets sold are tickets B, what percent of the total revenue on tickets sales are from B?

Discussion Link

41. After every 45 minutes trip, a car will take a 15 minutes break. In highway, the car¡¯s rate is 60m/h, and in country road, the rate is 40m/h. In a 4 hours trip, the car can travel how many more miles in highway than in country road?

42. How many ways are possible to arrange A, O, L, L, and P with two "L"

Page 10: Mitul's 2007 GMAT Sep

being separated by at least one letter?

Discussion Link

43. If an=3a(n-1)-x, and a5=99, a3=27, x=?

Discussion Link

44. 32^5=2^x?

Discussion Link

45. If A and C represent the area and the circumference of a circle, respectively, and A:C=1:2root2, A=?

Discussion Link

46. Of a group of people, the ratio of number of women to men to children is 7 to 2 to 5. What is the total number of the people?

1). women + children=12

2). number of men is less than 4

Discussion Link

47. Of the people in a banquet, if 75% chose dissert, what percent chose coffee?

1). 60% of the people who chose dissert also chose coffee.

2). 90% of the people who chose coffee also chose dissert.

Discussion Link

48. When integer n is divided by 3, the remainder is 2. Is the integer divisible by 5?

1). When divided by 45, the remainder is 30--the conditions seems incorrect.

2). The integer is divisible by 2

Discussion Link

49. In the xy-plane, both line K and L intersect with axis-y. Is K's intercept with axis-y greater than that of line L?

1). K's intercept with axis-x is greater than that of L.

2). K and L have the same slope.

Page 11: Mitul's 2007 GMAT Sep

Discussion Link

50. The range of set A and B combined is 600. If range of A is 200, what is the range of B?

1). ...

2). The least number in B is 100 greater than the largest number in A

Discussion Link

51. Is y>0? The question is somewhat similar to Q5.

1). (x-y)/x > 0

2). (x^2-y^2)/x^2>0

Discussion Link

Edited by soft rock on 07 September 2006 at 11:29pm

Back to Top  

soft rockGMAT Tutor

Joined: 20 October 2004British Virgin Islands

Posts: 1125Gender: Not Specified

Posted: 07 September 2006 at 11:31pm | IP Logged

52. If x<y<z and x, y, z are integers, which of the following must be greater than y^x+y^z?

1). x ^ (y+z)

2). y^2z

Discussion Link

53. Three machines can finish a certain work in 36 hours at the same rate. If a 4th machine with the same rate is added in, what is the time to finish the work?

Discussion Link

54. F(x)=1+1/x, F(F(x))=?

Discussion Link

55. S is the set of all solutions of equation (x-1)(x+2)+(x-1)(x+4)=0. Which of the following must be true?

Discussion Link

56. (r^2-s^2)^2=?

Page 12: Mitul's 2007 GMAT Sep

1). 4r^2s^2=120

2). (r^2+s^2)^2=4r^2s^2

Discussion Link

57. If a and b are none negative integer, ab=?

1). 4^a=2^b

2). 2^a=3^b

Discussion Link

58. What is the remainder when 9^845 is divided by 10?

Discussion Link

Discussion Link2

59. Two buildings are 10m and 12m high, respectively. If the distance between the bottoms is 6, what is the distance between two building tops?

Discussion Link

Edited by GMATer on 08 September 2006 at 9:06pm

Back to Top  

soft rockGMAT Tutor

Joined: 20 October 2004British Virgin Islands

Posts: 1125Gender: Not Specified

Posted: 08 September 2006 at 5:50am | IP Logged

60. ----P---R----M----S-----

In the number line, R is zero, M is midpoint between P and S, and PR=1/3PS. What is P?

1). M is 1.5

2). S is 6

Discussion Link

61. The average of 20 numbers is 82. How many numbers are equal to 75?

1). Three numbers are between 75 and 82

2). The median is 85

The question seems incomplete.

Page 13: Mitul's 2007 GMAT Sep

Discussion Link

62. n=?

1). The tens¡¯ digit of 11^n is 4

2). The hundreds¡¯ digit of 5^n is 6

Discussion Link

63. With a speed of 20m/h, a truck will passes through a 4024 feet bridge in approximate how many minutes?

Discussion Link

64. When integer n is divided by 9, the remainder is 3. Is the integer divisible by 5?

1). When n is divided by 45, the remainder is 30

2). n is divisible by 2

Discussion Link

65. In the figure shown above, ABCD is a rectangle, CD=2, OCD is an isosceles triangle. Is OCD an equilateral triangle?

1). Angle ADO=30 degrees

2). The area of ABCD is 2 root 3

Discussion Link

Page 14: Mitul's 2007 GMAT Sep

66. In the figure shown above, ABC is a right triangle, and three rectangles are square. What is the area of the largest square?

1). The sum of area of two small squares is given.

2). The area of the triangle is given.

Discussion Link

67. Each of the 50 people chose one dish from A, B, and C, where rates of A, B, and C are $15, $12, and $18, respectively. If the total cost was $480, how many people chose C?

1). The number of people who chose A was 12 greater than number of people who chose B.

2). The number of people who chose A was two times the number of people who chose B.

Discussion Link

68. Is X^2 < X^3?

1). X<X^2

2). X<1

Discussion Link

Edited by GMATer on 08 September 2006 at 9:12pm

Back to Top  

soft rockGMAT Tutor

Joined: 20 October 2004British Virgin Islands

Posts: 1125Gender: Not Specified

Posted: 11 September 2006 at 10:48am | IP Logged

69. The sales price of a certain item is D, and the sales tax is P percent. If someone paid $20 for the item and got change of C, is the sales price greater than $15?1). C<52). P=6

Page 15: Mitul's 2007 GMAT Sep

Discussion Link

70. Is 3x+1 divisible by 10?1) x = 4n+2 2) x > 4

Discussion Link

71. Is 2^[(x/y)x]<1? 1). x < 0 2). y < 0

Discussion Link

72. One unit of fat contains 9 calorie, one units of B contains 4 calorie, one unit of C contains 4 calorie. If a snack consists of 15 fat, 16 B, and 4 C, what percent of the calorie contained by the snack fat is provided by the fat?

Discussion Link

73. A people walked from signpost 1 to signpost 2 at the average rate of 0.125km/m. Is the distance between two signposts greater than 0.8km?1). It cost more than 400 second for the people finish the trip2). It cost less than 450 second for the people finish the trip.

Discussion Link

74. In a given square T we inscribe a circle S. Into that circle we inscribe a square R. Is the circumference of S greater than 10? 1). The side of square R is greater than 22). The side of square T is greater then 4

Discussion Link

75. The greater integer less than -|root3 - 3| is:

Discussion Link

Edited by GMATer on 11 September 2006 at 9:22pm

Back to Top  

soft rockGMAT Tutor

Joined: 20 October 2004British Virgin Islands

Posts: 1125Gender: Not Specified

Posted: 12 September 2006 at 5:39am | IP Logged

76. Each of the 45 books is either British or Spanish, either paperback or hardback. If a book is selected at random, is the probability that a paperback Spanish book will be selected greater than 1/2?

1). 30 books are paperback book.

2). 15 books are Spanish book.

Page 16: Mitul's 2007 GMAT Sep

Discussion Link

77. What is the remainder when n is divided by 10?

1). The tens?digit of 11^n is 4

2). The hundreds?digit of 5^n is 6

Discussion Link

78. If m and n are none negative integers, mn=?

1). 4^m=2^n

2). 3^m=2^n

Discussion Link

79. As the figure shows, the three identical circles are tangent to each other. If the area of the blue region is 64root3-32pi, what is the radius of the circle?

Discussion Link

80. If the area of the region encircled by axis-x, axis-y, and y=8x+a is greater than 1, what is the range of a?

Answer:

The absolute value of a is greater than 4

Discussion Link

81. Of the members in an organization, 30% are female, and 3/8 are married male. Which of the following is true?

I. The number of the female is greater than the number of the married male

II. The number of the female is greater than the number of the un-married male.

Page 17: Mitul's 2007 GMAT Sep

III. The number of the married male is greater the number of the un-married male.

Discussion Link

82. The surface of a Clock (in a dart game) is evenly divided to eight parts numbered with numbers 1 to 8. If we throw arrows three times, how many ways are possible that the total score is 16?

42, 56, 64?/P>

The game seems a Roulette game rather than a Dart game. Someone rotated the roulette three times, how many ways are possible that the total score is 16?

Discussion Link

83. When k^4 is divided by 32, the remainder is 0. Which of the following could be the remainder when k is divided by 32?

2, 4, 6?/P>

Discussion Link

Edited by GMATer on 13 September 2006 at 7:30am

Back to Top  

soft rockGMAT Tutor

Joined: 20 October 2004British Virgin Islands

Posts: 1125Gender: Not Specified

Posted: 12 September 2006 at 11:21pm | IP Logged

84. "I" is a positive number. If it is rounded to integer N, E(I)=(N-I)/I. For an instance, if I=1.5, then E(1.5)=(2-1.5)/1.5=33.33%. E(1.6)-E(2.5)=?

Answer: 5%

85. A number of 5,000 pots are placed in a 25,000 square yard field. Averagely, each pot occupy how many square feet? 1 yard = 3 feet.

A 15 b 45

Discussion Link

86. Is x>1?

1). x^2<x

2). x<rootx

87.Is X>Y ?

1). X+Y>0

Page 18: Mitul's 2007 GMAT Sep

2). Y^X<0

Discussion Link

88. An=An-1 +5, A5=31. A1=?

Answer:11

89. Which is more economic, copying or buying a $12.5 book with K pages?

1). K> 85

2). Copying a book with K+15 pages will cost more than $14

Discussion Link

90. Tom has x books and he can completely place them in the bookshelf by placing 10 copies in each line. If 10 more books are added in, he can completely place them in the bookshelf by placing 12 copies in each line. X=?

1). x<96

2). x>24

Discussion Link

Edited by GMATer on 13 September 2006 at 7:38am

Back to Top  

soft rockGMAT Tutor

Joined: 20 October 2004British Virgin Islands

Posts: 1125Gender: Not Specified

Posted: 13 September 2006 at 5:58am | IP Logged

91. A and P took three tests. A's score in the first and second test is 10 and 4 higher than B's, respectively. If P's average score for the three tests is 3 higher than that of A, in the third test, P's score is how much higher than that of A?

Discussion Link

92. A, B, C, and D took a certain test, if the sum of their scores is 400, is score of B greater than that of C?

1). A+B=100+C+D

2). A+C=80+B+D

The data maybe inaccurate. Author's answer is C

Discussion Link

Page 19: Mitul's 2007 GMAT Sep

93. What is the remainder when 43^43+ 33^33 is divided by 10?

Discussion Link

94. Sequence A consists of 10 consecutive odd numbers and B consists of 5 consecutive even numbers. If the least number in A is 7 greater than the least number in B, the average of the numbers in A is how much greater than average of numbers in B?

Discussion Link

95. If r, s, and t are integer, is r^3 *s*t^4 negative?

1) r*t is negative

2) s is negative

Discussion Link

96. S=1/32 + 1/33 + ... +1/64, S is in which of the following range?

0.5<S<1

97. One tank has a capacity of 64800 gallon, one gallon water weights 8.3 kg. A pile can fill the tank with a constant speed in12 hours, what is the weight of the water filled into the tank per min?

Answer: 64800*8.3/(12*60)

Discussion Link

98. If mn>0, whether m^3n^2>0?

1). m>0

2). n>0

Discussion Link

99. Is x^2+y^2 divisible by 5?

1). When x-y is divided by 5, the remainder is 1

2). When x+y is divided by 5, the remainder is 3

Discussion Link

100. R and S are integers, is R/S a finite decimal?

1). R is the factor of 100

2). S is the factor of 100

Page 20: Mitul's 2007 GMAT Sep

Discussion Link

101. A and B are integers, is A*B even?

1). A+B is odd

2). A is even.

Discussion Link

Edited by GMATer on 14 September 2006 at 4:05am

Back to Top  

soft rockGMAT Tutor

Joined: 20 October 2004British Virgin Islands

Posts: 1125Gender: Not Specified

Posted: 13 September 2006 at 10:31am | IP Logged

102. If 56<x<66, where x is an integer. x=?1). When x is divided by 2, the remainder is 12). x+1 is divisible by 3.

Discussion Link

103. The figure above shows the shape of a mirror. The mirror has a semicircle in each end of a rectangle. If the rectangle is 9 long and the ratio of the area of the rectangle to area of two semicircles is 9 to pi, what is the width of the rectangle?

Discussion Link

104. A rope consists of 40% of A and other matters. A certain time later, 50% of A were lost and the other remained. Now, the rope consists what percent of A?

Discussion Link

105. If X/Y=3/5, and X+600/Y+500=4/5, X+600 is how much less than Y+500?

Discussion Link

106. Which of the following is terminating decimal?I.1/12 II.1/10^2

Page 21: Mitul's 2007 GMAT Sep

III.1/2^10

Discussion Link

107. When a positive integer is divided by 4, the remainder is r; when divided by 9, the remainder is R. What is the greatest possible value of r^2+R? [Uncertain]23, 21, 17, 13, 11

Discussion Link

108. 35 percent of a number is decreased by 15, and the result is 25 percent of the original number. What is the value of the number?Author's answer is 1.5 The question seems incorrect.

Discussion Link

109. 2x^2+5x=12, what is the difference between the bigger root and the smaller root?

Discussion Link

Edited by soft rock on 20 September 2006 at 9:29pm

Back to Top  

soft rockGMAT Tutor

Joined: 20 October 2004British Virgin Islands

Posts: 1125Gender: Not Specified

Posted: 13 September 2006 at 11:36pm | IP Logged

Number of Times

Number of people

0 31 22 103 34 55 3

110. The table above shows number of people corresponding to the number of the times. What is the median of the number of times that at least is one?

Answer:

The number of people with at least one times is 23, then the median is the 12th people. The corresponding times is 2

Discussion Link

111. Is number 3 the tens?digit of x?

Page 22: Mitul's 2007 GMAT Sep

1). When x is divided by 100, the remainder is 30

2). When x is divided by 110, the remainder is 30

Discussion Link

112. How many prime factors does x have?

1). X is factor of 7200

2). 180 is factor of x

Discussion Link

113. x/3 is between 3 and 100, and it is the square of a prime number. How many such x are possible?

Answer: 3<x/3=k^2<100. k is prime number. So, root3<k<10. k could be 2, 3, 5, 7

Discussion Link

114. How many numbers between 1 and 100 are not divisible by 2 and 3?

30 31 32 33 34

Discussion Link

115. The height median of the 5 children in family A is 118 cm. The boy in family B is 128cm. How many children in A are taller than the boy in B?

1). The average height of children in A is 120cm.

2). The second height in A is 130cm.

Discussion Link

Edited by GMATer on 14 September 2006 at 4:09am

Back to Top  

soft rockGMAT Tutor

Joined: 20 October 2004British Virgin Islands

Posts: 1125Gender: Not Specified

Posted: 14 September 2006 at 10:06pm | IP Logged

116. If r and s are root of x^2+bx+c=0, rs<0?

1). b<0

2). c<0

Discussion Link

Page 23: Mitul's 2007 GMAT Sep

117. (x+2)(x+3)/(x-2)>0, x is integer. If x<5, how many x are possible?

Discussion Link

118. Set I is defined such that, 1). if x is in the set, -x also is in the set. 2). if x and y are in the set, then xy is in the set. Is 12 in the set?

1). 2 is in the set

2). -3 is in the set

Discussion Link

119. How many sides does a polygon have?

1). All the sides have the same length

2). The sum of all interior angles is 1440

Discussion Link

120. x is between 90/245 and 3/4.Which of the following could be x?

1/2; some fraction equal to 0.743; some fraction coming to0.367

Discussion Link

121. Mary goes from a to b and then from b to c. Speed for a to b is 40 feet/min and speed for b to c is 60feet /min. She took 3 more hrs to go from a to b and the distance between a and b was 90 miles more. What was the time taken for b to c?

1. she took less than 45 mins

2. she took more than 42 mins.

Discussion Link 

122. A disc contains 10 songs. How many tracks do the songs occupy?

1). each song averagely occupies ...tracks

2). Median is ...

Discussion Link

123. The number of rooms of hotel G is 10 less than twice of that of H, and two hotels totally have 425 rooms. How many rooms does G have?

Discussion Link

Edited by soft rock on 20 September 2006 at 9:17am

Page 24: Mitul's 2007 GMAT Sep

Back to Top  

soft rockGMAT Tutor

Joined: 20 October 2004British Virgin Islands

Posts: 1125Gender: Not Specified

Posted: 16 September 2006 at 10:32pm | IP Logged

124. 34/67<x< 151/200, x could be which of the following numbers?

I. 1/2

II. 35/67

III. 150/201

Discussion Link

125. Integers x and y have three digits and z is the sum of x and y. Is the tens?digit of z equal to the sum of the tens?digits of x and y?

1). Both x and y have units?digits greater than 6

2). The sum of tens?digit of x and y is 7

Discussion Link

126. Sequence a1, a2, a3, ...an, after the first term, each term is equal to the previous term minus the following term. a7=?

1). a5-a8=...

2). a6=a8+...

127. Is x+3y even?

1). x-3y is odd

2). x-y is odd

Discussion Link

128. A person makes a sit-up exercise plan. First day, he will do five sit-up, then 6 in the second day, 7 in the third day...How many sit-up will he do in the following 16 days?

Answer: a1=5, d=1, n=16, S16=na1+n(n-1)/2 d =200

Discussion Link

129. Point (r,s) lies on the line L. Is the line's intercept with axis-x greater than r?

1). L has a negative slope.

2). s<0

Page 25: Mitul's 2007 GMAT Sep

Discussion Link

130. X=10^n*25^2, what is the unit's digit of x?

1). Forget...useless

2). n^2=1

Answer is E

Discussion Link

131. 1/root2-root50=?

-9/2*root2

Discussion Link

132. If x is a prime number, is 3xy even?

1). x>2

2). y is odd

Discussion Link

133. If b is an integer and b<10, x=1+ b/100. b=?

1). 1<=b<=3

2). The thousandth's digit of 10X^2 is equal to the tens's digit of x^2

Discussion Link

134. Of the 24 positive integers, all have the units's digit of 5, 1/3 have tens?digit of 0, 1/3 have tens' digit of 1, 1/3 of tens' digit of 2. What is the tens' digit of sum of 24 numbers?

Discussion Link

Edited by soft rock on 20 September 2006 at 9:14am

Back to Top  

soft rockGMAT Tutor

Joined: 20 October 2004British Virgin Islands

Posts: 1125Gender: Not Specified

Posted: 17 September 2006 at 5:01am | IP Logged

135. The revenue of a certain company increased x percent from 1997 to 1998, and decreased x percent from 1998 to 1999. If the revenue decreased 9% from 1997 to 1999, x=?

Page 26: Mitul's 2007 GMAT Sep

Answer: x=3

Discussion Link

136. Six years ago, a person deposited a certain amount of money in account A at 3% simple annual interest rate, and other amount in account B at 4% simple annual interest rate, respectively. If there was no any withdraw or deposit during the six years, is the current interest in A greater than that in B?

1). The amount invested in A was $1000 more than that in B

2). The investment in account A obtained an interest of $10 last year.

Discussion Link

137. The ratio of the number of registered publican to the number of registered democrats is 3/5. If 600 Publican and 500 Democrats were added in, the ratio became 4/5. What is the difference between the two current numbers?

Answer: 300

Discussion Link

138. Set S and T have 100 numbers, respectively. Is standard deviation of S greater than or less than that of T?

1). Range of S is greater that of T

2). Average of S is greater than that of T

Discussion Link

139. In a certain test, how many of the 35 students in a certain class got a score less than 85?

1). The average score of all students is 85

2). The median score of all students is 85

Discussion Link

140. Is the tens?digit of x greater than that of y?

1). x-y=37

2). The units' digit of x is ... greater than that of y

Discussion Link

141. X+Y=?

Page 27: Mitul's 2007 GMAT Sep

1). x/y=2

2). y^2=4

Discussion Link

142. uv>0,is (u^2)(v^3)>0?

1). u>0

2). v<0

Discussion Link

143. A company has 10 male and 15 female employees. If 15 people drive to office and 10 people, including 8 female take bus, how many male drive to office?

Discussion Link

144. A car has a 15 gallon-tank. With one gallon gasoline, the car can travel 30 miles when driving in the city, 20miles in the countryside. What expression is the fraction of the full fuel tank of a car uses when driving 60 miles in city and 30 miles in the country?

Answer: (60/30+30/20) /15

Discussion Link

Edited by soft rock on 20 September 2006 at 9:20am

Back to Top  

soft rockGMAT Tutor

Joined: 20 October 2004British Virgin Islands

Posts: 1125Gender: Not Specified

Posted: 18 September 2006 at 6:01am | IP Logged

145. If P is a prime number greater than 2, which of the following could be a prime number?

2p, p^2, p+2, p/2, (p-1)/p

Discussion Link

146. If 300<X<400, is the tens?digit of x greater than 5?

1). The units' digit of x is greater than 4

2). When x is added with 237, the hundreds?digit will be equal to 6

Discussion Link

147. An equipment cost $10,000. After the first year, 80% of the value of the previous year value will be remained. 9 years later, how much will be

Page 28: Mitul's 2007 GMAT Sep

the equipment value, in thousand dollars?

Answer: 10(0.8)^9

Discussion Link

148. A, B, and C are the sides of a triangle. If A=4, B=3, which of the following can express the value of C?

Answer: 1<C<7

149. x<y<z, where x, y, and z are positive integer, which of the following must greater than (y^x)(y^z)?

Answer: y^2z

Discussion Link

150. K is a positive integer. If the sum of integers from -15 to K, inclusive is 51, K=?

Discussion Link

Edited by GMATer on 20 September 2006 at 9:24pm

Back to Top  

soft rockGMAT Tutor

Joined: 20 October 2004British Virgin Islands

Posts: 1125Gender: Not Specified

Posted: 20 September 2006 at 4:51am | IP Logged

151. A square brick with area of 9 inches is 1/8 inches thick. A number of bricks are spread in a line, and continues a length of 144 inches. If the bricks were stacked one by one, what is the height?

Answer: 6

Discussion Link

152. Which of lines R and L has a greater intercept with axis-y?

1). R has a greater intercept with axis-x than L.

2). R is parallel line L

Discussion Link

Discussion Link2

153. Tax amount = taxable income * tax rate (not mentioned that the rate is fixed)=(total income- some deductions)* tax rate.

A person's taxable income = total income - some deductions, and one of the deduction is donation. If his donation resulted in a $217 tax

Page 29: Mitul's 2007 GMAT Sep

decreasing, how much did he donate?

1). the tax rate for the portion of taxable income that is over than 7500 is 28%

2). His taxable income is 7753

Author's answer is C

Discussion Link

154. A person's income consists of wage and deduction. This month, his wage is 180% of the deduction. The deduction is what percent of the total income?

Discussion Link

155. A hiker walking at a constant rate of 4 miles per hour is passed by a cyclist traveling in the same direction along the same path at a constant rate of 20 miles per hour. The cyclist stops to wait for the hiker 5 minutes after passing her, while the hiker continue to walk at her constant rate. How many minutes must the cyclist wait until the hiker catches up?

Discussion Link

156. Is line A's slope less than B's slope?

1). A and B intersect at point P (10,1)

2). the y-intercept of A is greater than that of B.

Discussion Link

157. Students in the cafe, each either like tea or dislike tea, each either like coffee or dislike coffee. If 2/3 students dislike tea, and in those students dislike tea, 3/4 dislike coffee. How many students like coffee?

1). Total number of the students is 120

2). 40 students like tea

Discussion Link

158. A company get a 50000 dollar contract of a project, the project only cost labor and material, is the profit larger than 15000?

1).Labor cost is twice the cost of material

2).Profit is larger than Labor cost

Discussion Link

Discussion Link2

Page 30: Mitul's 2007 GMAT Sep

Edited by GMATer on 20 September 2006 at 9:22pm

Back to Top  

soft rockGMAT Tutor

Joined: 20 October 2004British Virgin Islands

Posts: 1125Gender: Not Specified

Posted: 20 September 2006 at 9:10am | IP Logged

159. What is the unit's digit of X?1). x/(10^n)=25^22). n^2=1

Discussion Link

160. Both B and W's wage are greater than 50,000. Is W>B?1). B is closer to 50,000 than W2). The difference between B and 35,000 is less than the difference between B and W

Discussion Link

161. x and y are 2-digt integers. What is the difference between two tens' digit?1). x-y=272). Units' digit of x minus the units' digit of y is greater than 3

Discussion Link

162. Each of ten items was cost C dollars. If 8 items were sold out at price of D (d>c) and 2 items were sold out at price of 1.1C, what is the ratio of profit to cost?1).d=c+82).d=1.2c

Discussion Link

Discussion Link2

163. Is X^3>y^2   1). y=x^22). x>x^2

Discussion Link

164. Two people are working on a equation x^2+bx+c=0, the first people get two results x1=... and x2=..., but the numbers are incorrect, they are the result for x^2+bx+d=0; the other people get two results x1=... and x2=...,but the numbers are incorrect, they are the result for x^2+ex+c=0. What is the correct solution of equation x^2+bx+c=0?

Discussion Link

Edited by GMATer on 20 September 2006 at 9:00pm

Back to Top  

Page 31: Mitul's 2007 GMAT Sep

soft rockGMAT Tutor

Joined: 20 October 2004British Virgin Islands

Posts: 1125Gender: Not Specified

Posted: 24 September 2006 at 10:22am | IP Logged

165. Given 452*37*16, add 1 to which one of the 7 digits will increase the value of the product by less than 1000?

Discussion Link

166. Is x greater than 13/11?1). X>1.1822). X<1.19

Discussion Link

167. y is not zero, is (2^x/y)^x<1?1). x>y2). y< 0

Discussion Link

168. X+Y = 1; X^2 + Y^2 = 3, XY=?

Discussion Link

169. A certain number of products A were sold out and the total cost was 5 percent of the sales value plus $100,000. If the sales made a profit, is the number the products greater than 21,000?1). The total sales value is $110,0002). The price of the product is $5Answer:5X > 100,000 + 0.05*5X => X > 100,000/4.75 > 21,000, answer is b.

Discussion Link

170. In the number line, is point P between Q and Z?1). The product of P and Q is greater than 02). The product of P and Z is less than 0

Discussion Link

171. In triangle ABC, P is a point on BC. Is AP=AB?1) AB=BP2). Angel APB is 60 degree

Discussion Link

172. Symbol * is denoted to be X*=1/X, then (3*+2*)*=?

Discussion Link

173. The median of 11, 18, 43, 60 and x is 18. What is the greatest possible average of the five numbers?

Discussion Link

174. If the 5 times of the age of A is 10 less than 3 times of age of B, what

Page 32: Mitul's 2007 GMAT Sep

is age of A?1). The sum of two ages is 14.2). A is ten year younger than B.

Discussion Link

175. X=?1). 4^(xy) = 2^(6x)2). 9^(y) = 3^(xy)

Discussion Link

Edited by GMATer on 24 September 2006 at 9:42pm

Back to Top  

soft rockGMAT Tutor

Joined: 20 October 2004British Virgin Islands

Posts: 1125Gender: Not Specified

Posted: 26 September 2006 at 8:42am | IP Logged

186. Each of 25 fields can produce 10 tomatoes, if farmers produce x additional fields, each field's production will reduce by x tomatoes. If farmers produce more than 25 fields and the total production is 174 tomatoes. How many fields did the farmers produce? Answer: 29 (25+x)(10-x)=174, x=4. So, 25+4=29

Discussion Link

187. Prime twins: if the difference between any two prime numbers is 2, we can call them a pair of prime twins. Which of the following can be the average of the sum of a pair of prime twins?Answer:30=(29+31)/2

Discussion Link

188. A tank is filled with gas or something like this, were there more than 30 gallons of gas in the tank before the tank went to leak? (Note 1 gallon=128 ounces)1). the leaking speed is 6.4 ounce per minute. 2). After it began to leak, the tank went empty less than 12 hours.

Discussion Link

Discussion Link2

189. x is in the rectangle OABC, D is one point on side BC, what is probability of x is in the triangle OAD? (Key: 1/2)

Discussion Link

190. What is the x-axis intercept of this line L? 1). L passes through point A (0,5)2). L passes through point A (5,4)

Page 33: Mitul's 2007 GMAT Sep

Discussion Link

191. In a class, 70% of the student choose geometry, 80% of the student choose physics, what is the least probability of students choose both geometry and physics?

Discussion Link

Discussion Link2

192. Is a-3b an even number? 1). b=3a+3 2). b-a is an odd number

Discussion Link

Discussion Link2

193. 4000 gallon can fill the capacity of a tank from 3/4 to 4/5, what is the full capacity of the tanker?

Discussion Link

194. A sequence of numbers is 1/2, 1/4, 1/8, 1/16, 1/32, what is the range of the tenth number a10 in the sequence? Answer: 0.0001<a10<0.001

Discussion Link

195. A, B, C and D play cards, the total score of A, B, C and D is 400, is B>C? 1). A+B=100+C+D 2). A+C=80+B+D

Discussion Link

Edited by GMATer on 26 September 2006 at 9:20pm

Back to Top  

soft rockGMAT Tutor

Joined: 20 October 2004British Virgin Islands

Posts: 1125Gender: Not Specified

Posted: 26 September 2006 at 9:54am | IP Logged

196. If X^2 + (1/x^2) = 4, x^4 +(1/x^4) = ?

Discussion Link

197. Is x+y even?1). X is multiple of 32). Y is multiple of 3

Discussion Link

198. 100<X<1000, does integer x have a digit greater than 5?

Page 34: Mitul's 2007 GMAT Sep

1). X>4502). X>550

Discussion Link

199. 75% of X is 24 and 120% of Y is 30, Y-X=?

Discussion Link

200. It takes the same time for a ship to travel 80 miles upstream trip and to travel 120 miles downstream. If the ship maintains a rate of 20 mile/h in still water, what is the speed of river water?

Discussion Link

201. A car traveled at a constant speed of 88 feet/second. Did it finish a 40 miles trip in T hours?(1mile=5280 feet)  1). T<45  2). T>42

Discussion Link

202. Set S and T each contains 100 numbers. Does S have a standard deviation greater than that of T?1). The average of S is greater than that of T2). The median of S is greater than that of T

Discussion Link

Edited by GMATer on 26 September 2006 at 9:09pm

Back to Top  

soft rockGMAT Tutor

Joined: 20 October 2004British Virgin Islands

Posts: 1125Gender: Not Specified

Posted: 27 September 2006 at 3:04am | IP Logged

203. 20 people take a test and the average score is 82. How many people have a score under 75?

1). exactly 3 people have score between 82 and 75.

2). the median score is 85.

Discussion Link

204. A car dealer sold total 21 cars with revenue of $252,000, 1/3 of them are used cars. And, the price for the used car is $9000 less than the new car. What is the revenue on the used cars sold?

Answer: 42,000

Discussion Link

Page 35: Mitul's 2007 GMAT Sep

205. A number of 140 cards are averagely distributed to m people. If m is no less than 2 and each people get at least 2 cards, how many "m" are possible?

Answer:

Given that 2<= m <= 70. Basically, you are looking for how many possible of m are there can be divided by 140. So, m could be 2, 7, 14...., 70. Total is 10.

Discussion Link

206. Points A and B are on the banks of a river. It takes a boat one hour to go downstream from A to B, and 1 hour 30 mins to go upstream from B to A. How long would it take the boat to go between points C and D on the shore of a lake if the distance between C and D is the same as that between A and B?

Answer:

Let x = distance A to B

Let u = speed of current of the river

Let v = speed of boat in still water.

Now we have

x/(v+u) = 1

x/(v-u) = 1.5

At last we can get that 2.5x = 3v

Then x/v = 3/2.5= 1.2 hours

Discussion Link

08.05

 ScoreTop:VIP Only Section:08.05

Topic: August Math Answers--Explanations(31/08)

Forum Jump  

Author Message Prev Topic | Next Topic 

Page 36: Mitul's 2007 GMAT Sep

SpidermanGMAT Tutor

Joined: 02 May 2005India

Posts: 945Gender: Not Specified

Posted: 04 August 2005 at 2:02pm | IP Logged

21.It takes four hours for A to manufacture 1/4 of a product, 2 hours for B to manufacture 1/6 of the product, 8 hours for C to manufacture 1/3 of the product. How much time will it take for A, B, and C working together to manufacture 24 such products?

Discussions: MJJ 19 -23

lets compute the product completed in 1 hour for each manufacture.

A = 1/16 , 8 = 1/12 , C = 1/24

A+B+C= product completed in 1 hour, which is

3+4+2/48 = 9/48....

So, in 1 hour , they together produce  9/48 of a product...

How many hours will they take to make 24 products?

Ans= 24 * 48/9 ~ 120 hrs

22.A, B, C, D, E, F are points on a number line. Whether AC>DF?1). AB<DE2). BC>EF

Discussions: MJJ 19 -23

E for me. Order of the points is not mentioned.

23.If a code word is defined to be a sequence of different letters chosen from the 10 letters A, B, C, D, E, F, G, H, I, and J, what is the ratio of the number of 5-letter code words to the number of 4-letter code words?A. 5 to 4B. 3 to 2C. 2 to 1D. 5 to 1E. 6 to 1

Discussions: MJJ 19 -23

Ans: different ways to choose  5 letters is 10P5       Different ways to shoose 4 letters is 10P4.

Ratio is  6 to 1.. E

24.If the cube root of Y is 2, Y^6=?

Discussions: MJJ 24-29

Page 37: Mitul's 2007 GMAT Sep

Obviously, Y  is 8.

Its 8 ^6.

25.If x*y=165 and x>y, which of the following is the least possible value of x-y?2, 4, 8, 23, 15

Discussions: MJJ 24-29

If X and Y are two positive integers, then its 15 *11 . hence, the difference of 4.

26.A and B had a dinner in the same restaurant. Both them used the 10 percent coupon and chose the same dishes. A paid 15% of the original price as tip and B paid 15% of the discounted price as tip, and A paid $0.63 more than B. What is the original price of the dishes?

Discussions: MJJ 24-29

Conside the bill amount to be 100.Discounted price is 90.

Tip on actual bill = 15Tip on the discounted bill = 13.5

Difference is 1.5hence, for 100 its 1.5, for ? its 0.63.its 63/1.5 = 42

27.The surface of a cube is ..., what is the side length?

Discussions: MJJ 24-29

Surface area is 6a^2, where a is one of the sides.

28.By increasing product's price, a game company up its profit from 20 percent of the cost to 25 percent of the cost. If the cost does not change and the new price is $80, what is the original price?

Discussions: MJJ 24-29

If x is the actual cost, 1.25x = 80, x = 64

29. E.

1)  and 2 ) Obviously not sufficient

Combining:  We have a st line from 3q and 4Q. This can go from either 2Q ( -ve slope)  or 1Q ( + slope).  Hence, answer is E.

30.If n and m are positive integers and m=n^3/1800, what is the least value of m?Answer: 151800=(3^2)(2^3)(5^2), n^3=(3^3)*(2^3)(5^3)=15*(3^2)*(2^3)(5^2),

Page 38: Mitul's 2007 GMAT Sep

so, m=15

Discussions MJJ30-48

31.If x and n are positive integers (or integers?), and when (n+1)(n-1) is divided by 24, the quotient is x and the remainder is r. r=?1) 2 is not the factor of n2) 3 is not the factor of n Answer: C

Discussions MJJ30-48

From the question we know that: x*24 + r = n^2 - 1,we want to know what r is: 1) We know that n = 2k + 1, then x + r/24 = (2k + 2)(2k)/24 => (k + 1)*k/12 => which we still don't know a darn thing about r, so peace to 1) by itself; 2) We know that n = 3k + 1 or 3k + 2, then if n = 3k + 1, then x + r/24 = (3k + 2)(3k) / 12 => no idea what r isIf n = 3k + 2, then x + r/24 = (k + 1)*(3k -1) / 8 => no clue what r is; so goodnight to 2) by itself1) and 2) together: now we know that n is either 6k + 1 or 6k + 5, if n = 6k + 1, then x + r/24 = (36k^2 + 12k) / 24 => (3k^2 + k)/2 => 3k(k + 1)/2, since either k or k+1 has to be even, and x is an integer, r has to be 0;If n = 6k + 5, then x + r/24 = (36k^2 + 60k + 24) / 24 => (3k^2 + 5k + 2)/2 => (3k + 2)(k + 1) / 2, since either 3k + 2 or k + 1 has to be even, and x is an integer r has to be 0;

Discussions MJJ30-48

32.If the cube root of y is 5, and y=root x, x=?

Answer: 125^2

Discussions MJJ30-48

33.S(n) is denoted that the number of the factors n has, for example S(120)=5, because 120=2*2*2*3*5. If S(m)=4, S(q)=12, S(mq)=?Answer: 16

Discussions MJJ30-48

34.Someone went to seashore at the rate of 50miles/h and returned at the rate of 38miles/h. If the one-way is 95 miles, how much time did he cost?

Time taken to get to the beach is 95/50 = 1.9 hrs

Time taken  to return from the beach = 95 /38 = 2.5 hr

Total Time = 4. 3 hrs = 4hrs and 18 minutes

Answer: 4 hour and 24 minutes.

Page 39: Mitul's 2007 GMAT Sep

Discussions MJJ30-48

35.Does a circle intersect with axis-x?1) The distance between the center and axis-x is greater than that between center and axis-y2) The circle intersects with axis-y

With 1=> Not possible. You can have case ranging from the circle touching the X-Axis as well as Y-Axis to not touching eighter one.

with 2=> Not possible

With 1 & 2=> Not possible, as said in case 1.

Hence, E.

Answer: E

Discussions MJJ30-48

36.5X-3Y= ?1) X/0.3-Y/0.5=...2) X/0.5-Y/0.3=...

A it is.

Please Note.. YOU DO NOT HAVE TO SOLVE THE DS PROBLEM TO ANSWER.

Answer: A

Discussions MJJ30-48

37.It takes x hours for A and B together to make 1,000 products. If it takes 5x/3 hours for B to finish the same work independently, how many hours does it take for A to finish the work independently?

3/5x + Y = 1/x

Y = 1/x - 3/5xY = 2/5x We want 1/Y = 5x/2.

USE PLUGGIN METHOD IN GMAT.. SAVES TIME

Answer: 5x/2Discussions MJJ30-48

38.N=P+Q+R, P, Q, R are consecutive positive integers. N=X*Y*Z, X, Y, Z, are positive consecutive integers. What is the remainder when N is divided by 5?1) When P is divided by 5, the remainder is 12) When X is divided by 5, the remainder is 1

From 1 => let P = 5M

Page 40: Mitul's 2007 GMAT Sep

Hence, N = 5M + 1 + 5M + 2 + 5M + 3 = 15M +6Thus, gives a remainder of 1.

From 2 = > in the same Way, using remainder theorem N = mod 6

Hence, a remainder of 1.

Answer: D

Discussions MJJ30-48

39.Line m and n are perpendicular to each other. Is the abscissa of their intersection -1?1) The slope of m 2) n passes through point (0,4)

From 1 => Not possible to determine the intersection point

From 2 = > Not possible to determine the point of intersection

Both => Not possibleAnswer: E

Discussions MJJ30-48

40.There are three kinds of bonus, each is $750,$1500,$5768, the company will give bonus to its employees. Each bonus has be to signed to at least one person. Now the company has the total bonus of 5xxxx. What is the minimum number of employees who can get the bonus?10, 11, 12, 13, 14

To give to min ppl, the amount of bonus given out should be max.

Hence, to complete formalities, we give each of 750 and 1500 to 2 ppl, which is 2250.

If we give out 5768 to nine ppl, it exceeds 50000.

Hence, the min ppl is 11.

Reference answer: 12 ( Not sure of xxxx)

Discussions MJJ30-48

41.There are 10 males who smoke in a group. The number of females who smoke is half of the nonsmoker of female. Is the number of nonsmoker of male larger than the number of females who smoke?1) 12 males don't smoke in the group2) 60% of nonsmokers are femalesAnswer: B         &a mp;a mp;a mp;a mp;a mp;a mp;a mp;a mp;a mp;a mp;a mp;a mp;a mp;a mp;a mp;n bsp;  ;         ; ; ; ;   ;       &a mp;a mp;a mp;n bsp;   ;   

From the question we know: Ms, Fs: male/female smokers, Mn/Fn: male/female nonsmokers, Ms = 10, Fs = 0.5Fn, and we want to know

Page 41: Mitul's 2007 GMAT Sep

whether Mn > Fs1) Mn = 12, then it's not enough to know how many female smokers there are, so 1) by itself is not sufficient;2) Fn = 60%*(Mn+Fn) => 2Fs = 60%*Mn +1.2*Fs since Fs = 0.5Fn => 0.8Fs = 0.6Mn => Mn > Fs so 2) by itself is sufficient

Discussions MJJ30-48

42.On the first day, a company sells orange beverage mixed with the same amount of orange juice and water. On the next day, the company adds the water twice as much as the amount of orange juice sold on the first day. If the company sells a glass of the beverage at $0.6 on the first day, how much is the price on the next day if  the company get the same revenue?I choose $0.4

Discusions: Same Post: At the end

43.X is hundredth digit of 0.0X. Y is thousandth digit of 0.00Y. What is the proximate maximum ratio of the two numbers?Reference key: If ask x:y, the answer is 9; if ask for 0.0x/ 0.00y, the answer is 90

Discussions MJJ30-48

44.In xy-plane, Y=ax^2+bx+c,does the graph intersect with X axis?1). a>02). c<0Answer: Cb^2-4ac>0

Discussions MJJ30-48

45.

r s tu v w x y z

Each of the letters in the table above represents one of the numbers 1, 2, or 3, and each of these numbers occurs exactly once in each row and exactly once in each column. What is the value of r?1) v + z = 62) s + t + u + x = 6

Statment I -- v = z =3 .

Statement II - Possible, since x and u, and s and t, cannot take the same value and they have to be 1 or 2. r =3

Answer =D

Given Answer: DDiscussions MJJ30-48 and at the End

46.P = sum of positive odd numbers which are < 50

Page 42: Mitul's 2007 GMAT Sep

Q = sum of positive even numbers which are < 50

Different ways to calculate P,

METHOD I ---- P = 25/2(1 + 49) = 625 ..PROGRESSIONMETHOD II ---- The square of any integer( +ve) can be expressed as the sum of consecutive odd numbers.. eg:= 1+3 = 4 = 2 ^2...

similarly.. 1+..50 = 25^2 = 625

Q = 24/2( 2+ 48 ) = 600

P- Q= 25

The result of P-Q =?Answer: 25

Discussions MJJ30-48

47.Ds m and n are integers, is m^n an integer?1) n^m is positive2) n^m is an integer

Not sure abt the Q, since, an integer raised to the power of an other integer is always an integer !!

Discussions MJJ30-48

Given Answer: E

48.If xy+x+y=34, (x+1)*(y+1)=?if we expand (x+1) *(y+1) = xy + x + y +1=34 +1Answer: 35

Discussions MJJ30-48

49.What is the sum of all the integers greater than -26 and less than 24?

The sum is -25 -24........ +22 +23.. All cancel out except -25-24 = -49Answer: -49

Discussions MJJ 49-58

50.If the revenue increased by 15% and the expenditure decreased by 10%, by how much percent did the profit increase?Reference answer: 28%The question is incomplete

Discussions MJJ 49-58

51.If 37/13=2+1/(13/x), x=?

Page 43: Mitul's 2007 GMAT Sep

Simplifyin we get, 26+x/13 = 37/13... Hence, x =11

Discussions MJJ 49-58

52.In a certain disc store, the rent consists of $3 plus $19.5 for each disc. How many discs did a person rent?1) The total rent is between ... and ...2) The total rent is between ... and ...

Question is incomplete...The Idea is to see if an integer can be determined from the given statements.

Reference key: With one of the statements, we can get an integral number.

53.Profits=beverage + snack + sanwichy, and beverage>snack>sanwichy, whether is profits>6.6million?1) Snack + sanwichy=4.5  2) Beverage + snack=...

1=> Not Sufficient.2=> Not Complete. If the sum is greater than 6.6, then B.

E.

Discussions MJJ 49-58

Given answer: A

54.What is the range of the numbers 2, 3, 4, 6, 7, 9, 10, 12, 20, and x?1) The mean is 92) x is the median

1=> When the mean is given, we can find the missing number and hence, we can find out the range.

2=> when the median is given, we can also find out the series and find the range.Hence, D

Discussions MJJ 49-58

55.There are some green balls and blue balls in a box. what is the total number of the balls?1) The number of the green balls is 1/3 of the number of the total balls2) If one green ball is taken out, the remaining green balls are 1/5 of the balls in the box.

1=> G= 1/3 * A2=> g-1 = i/5(a-1)

Can be solved only using both the equations...

hence, C

Page 44: Mitul's 2007 GMAT Sep

Discussions MJJ 49-58

56.M is a two digits number and the tens' digit is x and the units' digit is y, then M^2 is which of the following?

M = 10x + y

M^2 = (10x+y)^2 = 100x^2 + y^2 + 20xy

Discussions MJJ 49-58

Answer: 100x^2+20xy+y^2

57.Root12+root27=root 75

58.Someone plan to invest $20,000 for one year. There are two interest rates for him to select, 8.5 percent and 12.5 percent. If he want get an amount of $2,000 interest, he invest at most how much to the 8.5%-rate account?

Answer: 2000<=x*0.085+(20000-x)*0.125 so x=12500

Discussion: MJJ 58-68

59.On a yard sale, no any two goods have the same price. If a certain goods is the fifteenth expensive and the twentieth cheap, how many goods are there?

15+20-1=34Discussion: MJJ 58-68

Answer: 34

60.If three people are to be selected from eight people, who include Bob and Nancy, how many ways are possible that Bob, not Nancy is selected?

We have 6 ppl + Bob + Nancy. We drop nancy and have Bob.

We need to select 2 ppl from 6. 6C2 = 15Answer: 15Discussion: MJJ 58-68

61.Someone purchased stock X at the price x per share and stock Y at the price y per share. What is the average price of all the stocks he owned?1) The total value of x is ... and the total value of y is ...2) The number of x is 1.5 times the number of yAnswer: EDiscussion: MJJ 58-68

62.A line passes through the origin, does it pass through point (-3,-2)?1) It passes point (9,6)2) The slope of it is 2/3

Page 45: Mitul's 2007 GMAT Sep

Since, one of the points is already given, we only need eighter the slope or an other point.

Hence, D.Answer: DDiscussion: MJJ 58-68

63.If m and n are integers, what is the remainder when 5*10^n+m [or 5*10^(n+m)?] is divided by 3?1) m=12) n=10

The concept here is, whenever any power of 10 is divided by 3, you get a remainder of 1.

With 5 you get a remainder of 2.

hence, irrespective of the power of 10, the remainder for powers of 10 will be 1 and the answer for the above eqn is 2.

Hence, D.Discussion: MJJ 58-68

64.Six colours (red.black.white.orange.pink.yellow) can be used to decorate. If one or more can be used, how many ways are possible that white is used? Answer: 321+C1,5+C2,5+C3,5+C4,5+C5,5=32Discussion: MJJ 58-68

65.Is P a prime number?

1) P is odd2) P+2 is a prime numberAnswer: EFor example: 7, 11Discussion: MJJ 58-68

66.Tank X has 200 gallons water and water is being filled into at the rate of M gallon per minute. Tank Y has 500 gallons water and water is being drawn out at the rate of R gallon per minute. How many minutes later, two tanks have the equal amount of water?1. 300/M+R  2. 300/M-R 3. 700/m+R 4. 700/M-RAnswer: 1200+nM=500-nR, nM+nR=300, then n=300/M+RDiscussion: MJJ 58-68

67.X is in the set 2, 4, 8, 10, and x, what is x?1) The average of the numbers is equal to the median2) X is not 16Answer: E X maybe -4, 6, 16Discussion: MJJ 58-68

68.If x is a two-digit number less than 50 and x=R5, R=? (R is the tens'

Page 46: Mitul's 2007 GMAT Sep

digit of x)1) The thousands' digit of x^2 is 12) The hundreds' digit of x^2 is 2Answer: AX maybe is 15, 25, 35, 451) Only 35 is ok2) Both 15 and 35 are ok.Discussion: MJJ 58-68

69.A cuboid is 25 by 10 by 5, except the bottom, all the side faces are covered with paper. (the bottom maybe 10 by 5?) How much paper will be used?

Since, we are talking about wrapping the box, we have to calculate the surface area of the box except the bottom.

10*5 + 2(25*10+25*5) = 425

Discussion: MJJ 69 - 80

70.If xy is a two-digit number, and x is the tens' digit, y is the units' digit. What is the hundreds' digit of (xy)^2? 1) x=3 2) y=4

From A alone, we cannot predict. 30^2 = 900, 39 ^2 < 1600.Same for B.Hence, C.

Discussion: MJJ 69 - 8071.Of 10 students in a class, 4 learn only French, and the other students learn Germany or Spanish. If three students are to be selected, how many ways are possible that the group selected includes at least one French-learning student?

Calculating Numer of ways that 3 ppl can be selected is 10C3.Number of ways that No french student is selected is 6C3.Hence, the number of ways that atleast one french student is selected is 10C3 - 6C3. 

Discussion: MJJ 69 - 80

72.If 8.xxx * 10^(-1) + 7.xxx * 10^(-2) + 3.xxx * 10^(-3) + 2.xxx * 10^(-1)=N, the most left nonzero digit of N is which digit? Units' digit Tens' digit Hundreds' digit Thousands' digit Ten thousands' digit

After completing the sum, N = 0.8732. Hence, the units digit.

Unit's digit

Discussion: MJJ 69 - 80

73.If today is Saturday, on which day of the week does the day that 492 days later fall? I am not sure: should today be accounted in?

Page 47: Mitul's 2007 GMAT Sep

Since, today is a saturday, 365 days later or 52 weeks and 1 day, it is Sunday.Remaining days left to account are 492-365 = 127 days = (127/7) 18 weeks + 1 day , which means that it is Monday.

Monday

Discussion: MJJ 69 - 8074.Somebody invested $17,000 at x% simple annual interest rate and earned Z dollor interest at the end of a year. How much should he invest in an account, which has y% simple annual interest rate, so that he can earn the same interest at the same time? 1) X=2 2) Y=50%X

We need to relate the interest rates here. From A alone, we cannot relate the two.

From B, yes we can.

(B)

Discussion: MJJ 69 - 8075.If the speed of a car is 30miles/h, 44feet needs how many seconds?

1 feet = 0.30 meter.

44 feet = 0.3 *44 meter.

30 miles/hr = 30 * 1.6 *1000 / 3600 (m/sec)

time = 0.3*44 /(30 * 1.6*5/18 ) , 0.3 * 44 * 18/ 240 = 0.99 sec

0.99 sec

Discussion: MJJ 69 - 80

76.Y=4/5Z? 1) Y=0.8Z 2) Z=1.25Y

From A, yes we can determine the relation.From B, yes we can determine the ratio.Hence, (D)

Discussion: MJJ 69 - 80

77.If the sales value in July is x% more than that in June, and the sales value in August is y% more than that in July, how much percentage is the sales value in August more than that in June? 1) x+y=10 2) (1+x%)*(1+y%)=... [not sure]

From A, we cannot determine, since it is merely addition of %.From B, yes we can. Infact that is the formula we use to calculate the compounded interest !!

(B)

Discussion: MJJ 69 - 80

Page 48: Mitul's 2007 GMAT Sep

78.Five students took a test. Four of them got score 65, 70, 75, 80, respectively. The other student got the top score. If the average of the scores is two higher that the median, what is the unknown score?

here, the median is 75.Hence, the average is 77. 77 = (65+70 + 75 + 80 +x )/5

x=95

Discussion: MJJ 69 - 80               &nbs p;           &n bsp;                  & ; ; ; ; ; ; ; ; ; ; ; ; ; ; ; ;nbs p; &nbs p;           &n bsp;                   & ; ; ; ; ; ; ; ; ; ; ; ; ; ; ; ;nbs p;  &nbs p;           &n bsp;                    & ; ; ; ; ; ; ; ; ; ; ; ; ; ; ; ;nbs p; &nbs p;    79.Is 30 the factor of N? 1) 30 is the factor of N^2 2) 30 is the factor of 2N

If N is integer(info provided in Question) then the Answer is (A) If no info about whether N is integer or not then Answer is (C)

Discussion: MJJ 69 - 80

80.1/2+1/4+1/8+...+1/512=? A.0<S<1/2 B.1/2<S<1 C.1<S<3/2 D. 3/2<S<2 E. 2<S<5/2

This is geometric progression.

a= 1/2, r= 1/2 ,l = 1/512

Sum = (1/2 - 1/512)/(1/2) = 255*2/512 = 255/256 < 1.

(B)

Discussion: MJJ 69 - 8081.If x=(2/3)^(-2) y=(2/3)^2 z= -(2/3)^2, which of the following is right?Answer: x = (2/3)^(-2) = 1/(4/9) = 9/4, y = 4/9,and z < 0,thus x > y > z

82.z<y<x?1) z-x<y-x2) -x<-y

From I, its not possible.From II, its not possible.

From I and II, yes, we can determine the order.Since, x>y, and z-x <y-x, we know that z<y.Hence, we can determine the sequence.

Pluggin and check !!

Answer: C

83.Mark and Jim invested $1,000 respectively. In the end of one year, whose interest is greater?1) Mark invested at the 4 percent interest, compounded semi-annually.2) Jim invested at 2 percent interest, compounded quarterly.

Page 49: Mitul's 2007 GMAT Sep

from I alone, we cannot determine the amnt invested by JIM at a given interest.From II, we cannot tell the interest at which mark invested.From Both, yes, we can determine the interest for both.

Answer: C

84.Factory M can produce 3000 bulbs in S hours and N can produce 3000 bulbs in T hours, how many concrete hours will it take for M and N to produce 3000 bulbs together?1) T*S/(T+S)=...2) T is two more than S

From I, yes, we can determine the number, it rolls out to .. 1/T + 1/S. Since, 3000 is common for both, it can be ruled out.

From B alone, we cannot determine, since, we do not have a constant number to calculate the same.

Answer: A

85.Which of the following functions represents a line that has exact one point, both the abscissa and ordinate of which are integers?A. y=2xB. y=2x+5C. y=x+1/2D. y=x/30E. y=(root3)*x

I guess the question should be, which of these does not have a point with both integers.

In E, since we have root 3, which is not an integer, we cannot have an integer space.Answer: E

86.Sets F and G contain 20 positive integers, respectively. If the average of the numbers in F is greater than the greatest number of G, which of the following must be true?I. The average of F is greater than that of G.II. The median of F is greater than that of G.III. The range of G is greater than that of F.

I, yes, It is true.Since, the avg of a series cannot exceed the greatest number in a series.

From II,It not true.F = 6,7,8... G = 1,2,3.

From III, definetly not true.We cannot say. As we do not have information on the given 20 numbers.

Answer: only I

87.Of the 50 people subscribed newspapers, 36 subscribed at least one newspaper. How many people subscribed only one newspaper?1) 5 people subscribed three newspapers2) 17 people subscribed at least two newspapers

from I, we cannot tell the number of ppl who subscribe to 1 ppl alone.From II, yes, we can tell the number, which is 36 - 17 = 19.

Answer: B

Page 50: Mitul's 2007 GMAT Sep

88.Of a group of people donate for a organization, 1/4 donate less than $200, 2/3 donate from $200 to $1000, and others donate more than $1000. If the average value of the donation more than $200 is 360, what is the average value of the donations more than $1000?1) The average value of the donations less than $200 is 1802) The average value of the donations from $200 to $1000 is $540

From I alone, we cannot determine the avg, since, we do not have the avg from 200$ to  1000$.

From II, yes, we can determine.

We have the information that , the avg of donations more than $200 is 360.

Let x, be the number of ppl who donated money.Then the number of ppl who donated b/w 200 and 1000 is 2/3 x.and those beyond 1000 is 1 - (1/4+2/3). = 1/12x.

hence, we can calculate, (540*2/3x + A *1/4X) /2/3x + 1/4X = 360

Answer: B

89.The numbers maybe inaccurate, but it does not matter.Let x, y, z be the number of the people denote less than 200, 200 to 1000, more than 1000, respectively.So, x:y:z=3:8:1.not sure, what it is.

90.Points O, R, and T are on the number line. Is T between O and R?1) The distance from O and R is greater than that from O and T2) The distance from R and T is less than that from O and R

From I, we cannot determine the order .. i.e, TOR or OTR.From II, we cannot determine the same.

FROM I and II, yes we can determine the order.

Answer: C

91.Which of the following can satisfy the conditions: either two of the numbers have common divisor, and except 1, the three numbers have no common factor.  A. 12, 36, 18B. 21, 28, 12C. 17, 26, 28D. ...E. ...

Answer: B

92.If the volume of a cuboid is x and the length to width to height is 3 to 2 to 2, then the height is:Answer:

Since, this is a ratio problem, lets take the total, which  is 7.

The vol is 3/7Y * 2/7Y * 2/7Y = x, Y^3 = 7^3  * x/12. Hence, the height is (2/7)*7*((x/12)^(1/3))

93.A rectangle is 30 by 20, in feet. 1 feet =0.305 m. What is the area, in square meter?Area = 30 * 0.30 * 20 * 0.30 = 54

Answer: 54

Page 51: Mitul's 2007 GMAT Sep

94.If x and y are integers, 2x+y=12, and |y|=<12. How many y are possible?

Answer: 13The even numbers greater than or equal to -12 and less than or equal to 12.

95.If x and y are positive integers, (x+2)/(y+3)>(y+2)/(x+3)?1) x*y=22) x>yFrom I, we only get a quadratic equation and we cannot solve that.From II, yes we can calculate. Plugin!!

Answer: B

96.The range of the male in a team is 14 inches, and the range of the female in the team is 12 inches, what is the range of the team?1) The shortest of the man is three inches more than the tallest woman.2) The tallest man is 75 inches

From I, .. the sequnce is.. all the girls.. and then all the boys. Hence, the range adds up, which is 28.From II, No it is not possible.Hence, A.Answer: A

97.What is the average of x, y, and z?1) The average of x and y is 42) The average of y and z is 6         & amp; amp; amp; amp; amp; amp; amp; nbsp; ; ; ; ; ; ; ; ; ; ; ;  

From I, it is not possible to determine the avg of all the three numbers. Z is missin.From II, same as I.From I and II, not possible, since even the addition does not help.

Answer: E

98.The line k pass the two points (-4,-2) and (4,6) ,which line is perpendicular to line k and intersect in the point (4,6)?a, x-y-10=0b, x+y-10=0c, x+y-2=0

Firstly, if a line intersects at one of the point above, the point should satisfy the eqn. Only B does that.Also, the slope is -1 for the perpendicular line.Answer: B

99.15% of all trees planted in the F and not flowered.40 percent of trees planted in the F are flowered. What is the percent of trees planted in the F?a, 55%    b,25%   Answer: Ball trees=t, In F tree=f,and then not flowered in F=0.15t,flowered in F=0.4f ¨¤ 0.15t=0.06f, 0.06f:0.4f=0.15t:x, x=0.1, 0.1+0.15=0.25=25%I think what it's trying to say is: out of all trees planted WHICH DID NOT FLOWERED, trees in F which did not flower count as 15% OF THOSE; and 40% of F trees flowered, so what % is the number of trees planted in F overall?So 15% / 60% = 25%

100.On a certain road, 10 percent of the motorists exceed the posted speed limit and receive speeding tickets, but 20% of the motorists who

Page 52: Mitul's 2007 GMAT Sep

exceed the posted speed limit do not receive speeding tickets. What percent of the motorists on that road exceed the posted speed limit?

consider the number of motorists as 100. 10% = 10. They speed over and receive tickets. Also, 20% of these  ppl exceed speed limit but no ticket =2.Total = 8/100 =  12.5%

Answer:12.5%

101.What is the tens' digit and the units' digit of 123456789^2?Answer: 21123456789^2=(123456700+89)^2--->89^2

102.

Country        Chile Haiti China Japan American GBR Canada Luxemburg FrancePer capita       67   40    28       405   102        102 & ; ; ; ; ; ; ; ; ; ; ; ; ; ; ;nbs p; 268      334   & ; ; ; ; ; ; ; ; ; ; ; ; ; ; ;nbs p;     &nb sp;   554water con-sumptionAccording the table shown above, which of the following is right?I.  Japan is more than 10 times of HaitiII.  The median of last five countries is 102III. The per capita water consumption of china and Haiti is 34

I. Yes 405 > 10*40II Wrong...III Wrong..

Answer: Only I. Is right

103.If x, y and a are numbers, is x > y?1) x + a > x - a;2) ax > ay;

Clearly, from I,  2a > 0 or a > 0From II, x > y, if a> 0. Hence, combining both the statements, C.Answer : C

104.If d1, d2, d3, d4 are four different integers, whether is 0.d1d2+0.d3d4 greater than 1?1) The least number is 42) (0.d1d2)(0.d3d4)>0.5

From I, If 4 is the least number, the sum will definetly  cross 1. Hence is suffcient.

Page 53: Mitul's 2007 GMAT Sep

From II, yes, we can determine that the sum is greater than 1.For the product to exceed 0.5, the individual numbers should be atleast >0.6

Hence, the result is D.

105.n+k=0?1) nk=02) n-k=0

From I, we cannot determine the sume.From II, we still cannot determine.From I and II, yes we can determine. (n-k)^2 + 4*nk.Hence, C.

106.If the product of the three digits of a three-digit number is 144, what is the tens' digit of the least such number?

Represent 144 in single digits as 2*9*8. Hence, the least number has to be 289.Answer: 8

107.If N is an even number and E is an odd number, which of the following cannot be an integer?A. N/EB. E/NC. N/(E+1)D. (N-1)/EE. (N+2)/(E-1)

Division of an Odd number by an even number is not possible.

Answer: B

108.If six people are to be divided to three groups, each of which contains two people, how many was are possible?

109.O is the centre of a circle in a coordinate. The distance between O and axis-x is greater than or equal to L, the distance between O and axis-y is M. Does the circle intercept with axis-x?1) L>M2) It intercept with axis-y

From I, Not possible. No references to L and M.from II, Not Possible. Only the reference to M is known.From I and II, Not possible, since the distance to x-axis is not known.

Page 54: Mitul's 2007 GMAT Sep

hence, E.110.Someone cost x for something and the sell price is 10% more than x and final sold price is 10% less than sell price. What is percent difference from sold price to x?

let the cost price = 100. 10% more = 110. 10% less on it = 99.hence percentage difference = 1 %.

111.A and B use x hour to finish 1000 units and A use 5/3 x hour to finish 1000 units and how many hours, in x, will B use to finish 1000 units?

3/5x + 1/B = 1/x. 1/B = 1/x - 3/5x = 2/5x.  takes 5x/2 hrs.

112.99999^2-1^2=? Answer 10^5(10^5-2) (a-b)(a+b)

113.Water is pumping into a cylinder container at the rate of 1cube meter per minute. If the water level rises one meter per minute, what is the area of the bottom?

Since, the volume that flows in = 1cu.m, and height rise is 1meter. the surca area of the bottom = 1 m.sq

114.One thing cut into 3 pieces, one have length of ... and other have length of ..., ask the fraction of the longest one v.s. the shortest one.

115.The four sides of a parallelogram have the same length. What¡¯s the length of one of the diagonals?1) The side length is 22) The length of another diagonal is 2

From I, we do not know if the parallelogram is a rhombus or square.From II, same as I.

From I and II, yes we can find out the lenght of the rhombus . 2^2 = 0.5*2 * D. D= 4Answer: C

116.There are 10, 20, and 30 students in class P, Q and R, respectively. In a certain test, the average score of the students in

Page 55: Mitul's 2007 GMAT Sep

the class P, Q, and R are 79, 85, and 91, respectively. What's the average score of the total students?

10 *79 + 20 * 85 + 30 * 91 / 60Answer: 87

117.Which of the 1/x, x and x^2 is the least?1) x>02) x<1

From I , we cannot determine.Since, the fractional value  and the integral values do not concur.From II, -ve integers and their fractional values also have to be taken into acc.

From I and II, yes we can safely assume that they are fractional value and X is the least.

Answer: C

118.A and B need three hours and six hour to finish a certain work, respectively. How many hours will it take for them to finish half of the work together?

1/3 + 1/6 = 3/6 = 1/2 , 2 hoursAnswer: two hours

119.The function T=8h-3t represents the increased amount of the water in a certain container, where h is the inputting time and t is the outputting time during one day, in minute. What's the inputting time in a certain day?1) The increased amount of water is 2002) In the day, outputting time is 20 minutes

From I or II, we will have two unknowns in the eqn , hence cannot solve.With both I and II, we can determine the value.

Answer: C

120.A sequence is such that, T(n+1)=Tn/2, then T5=?1)T3=1/42) T1-T5=15/16

From I, yes, we can determine the the value of T5. that is t4 = 1/8 and t5 = 1/16.

From II, also we can determine the

Page 56: Mitul's 2007 GMAT Sep

value of t5, just by replacing T5 successively by the equation.t1 - t5=> t1 - t4/2 => t1 - t3/4.. and so on..

Answer: D

121.During 90 days, a certain beach sold tickets to the tourists. There are two kinds of tickets, daily ticket is $5 each and the seasonal ticket is $200 each. If the total amount of the revenue on the tickets is $11,000,000, how many seasonal tickets had been sold?1) ...2) A number of ... daily tickets had been sold.Answer: D

122.If n=3k, is k an integer?1) n is an integer2) n/6 is an integer

From I , it is not possible. N can also be a prime number and so on.From II, yes, we can determine. It says that n/6 is an integer. Hence, n/3 is also an integer.

Answer: B

123.If x and y are positive, then (x^1/n*y^1/n)^n=?Answer: xy

124.Letters A, B, C, C and D are used to compose a code, such as ABCCD, ABCDC. How many ways are possible that between two letters C, there are at least one another letter?12, 24, 36, 48, ...120 ! / 2 !

Answer: 36a, b, c, c, d: first get the combo. Between a b d => 3!, then x a x b x d x, then pick two out these four blank spots to insert c:Thus p(3, 3) * c(4, 2) = 6 * 6 = 36

125.DS: what is the average of a, b, and c?1) a + b + 3c = 17;2) a + b - c = 1;Answer: CLet (a+b+c)/3=v(average ), then 3v+2c=17, 3v-2c=1, v=3

126.DS: a number x = 5*(10^n) + m, m and n are positive integers and x / 3 has remainder r, what is r?1) n = 5

Page 57: Mitul's 2007 GMAT Sep

2) m = 1

Consider this part of the equation, 5 * (10 ^ n ). this will always lead to 50, 500.. and so on.

Hence, From II, the addition of 1 to 50, 500 or ... will lead to 51,501...From the divisibility rule, the numbers are divisible by 3.

Answer: B

127.Machine M can finish a job in 12 hours by itself;if machine M finishes 2/3 of the same job, and then machine N, whose work rate is 1/10 of that of machine M finishes the rest of the job, what is the total amount of time that took these two machines to finish the job?Answer: 48Let the rate of M to be 1/12, then rate of N, (1/12)*(1/10)=1/120(2/3) / (1/12)+ (1/3) / (1/120)=48hr

128.Is x > y?1) x^2 > y^22) y - y^2 > 0

From I, we cannot determine the sign on the variables. Hence, not possible.From II, we cannot determine if Y is positive or not.From I and II, still not possible.

Answer: E

129.If x, y, a are numbers, is x > y?1) x + a > x - a2) ax > ayAnswer: C1) x + a > x - a => 2a > 0 = > a > 0;2) ax > ay => x > y if a > 0, x < y if a < 0;1) and 2) together gives you x > y since 1) tells u a > 0

130.Some girl is buying apples and oranges; apples costs $0.4 a piece, where orange costs $0.6 a piece;if the girl bought 10 of apples and oranges, and the average cost is 56 cents, how many oranges does she have to put back in order to have an average cost of 52 cents for the remaining fruits?

0.4 ( x) + 0.6 ( 10 -x) / 10 = .56; -0.2x + 6 = 5.6;x = 2.

Page 58: Mitul's 2007 GMAT Sep

Now, 0.4 * 2 + 0.6 (8-x)/10 -x = 0.52; 0.8 + 4.8 -0.6x = 5.2  - 0.52x;0.4 = 0.08x; x = 5

Answer: 5

131.A guy sent an order of sofas and loveseats, if the total cost was $3800, how many sofas did he buy?1) One sofa costs $800 and one loveseat costs $300.2) There was more than one sofa in the order.

Not sure of the answer.From I alone we can determine the answer, becuase without involving both the terms the sum does not end up to 3800.

My answer : A.

Given Answer: C

132.A tax of 6 percent is charged to adult clothing, and no tax is charged for kids clothing, if a dude bought $149.00 of clothing, how much did he spend on kids clothing?1) The kids clothing cost 43 dollars more than adults clothing2) If the same percent of tax is charged for kids clothing, the total would be ...Answer: D1) Let k denote how much he spent on kids clothing, and a be how much he spent on adult clothing, then 1) gives the following:k + 1.06a = 149k - 1.06a = 43 => 2k = 192 => k = 962) k + 1.06a=149, 1.06k+1.06a=...

133.Two squares x and y, what's the ratio of the perimeter of x to the perimeter of y1) One side of x is a diagonal of y2) The ratio of one side of x to one side of y is square root of 2

From I, we know that if side of square is x. the diagonal is x*root2. Hence, we can determine the ratio.From II, yes we can determine.Answer: D

134.You have a rectangle of size 15 feet by 12 feet, if we want to fit it with squares with a side of 9 inches, how many we can fit in

Page 59: Mitul's 2007 GMAT Sep

such rectangle?(1 foot = 12 inches)Answer: 320, (15*12)*(12*12)/9*9=20*16

135.Someone invested at annual interest rate r, compound quarterly. What's the value of r?1) At the end of the year, the amount of the interest is between $800 and $850.2) The interest earned in the second quarter is $4 more than that in the first quarter.

From I, it is not possible to determine the value of r.From II, we cannot find out the rate, since we do not know the principle amnt.

My Answer : E

Given Answer: B

136.One apple is 40 cents, and one orange is 60 cents. Now J bought total 10 apples and oranges and the average mean is 56 cent. If J wants to keep the average mean 52c, how many oranges should be taken out?My answer is 5.

137.Yesterday each of the 35 members of a certain task force spent some time working on project P.  The graph shows the number of hours and the number of members who spent that number of hours working on project P yesterday.  What was the median number of hours that the members of the task force spent working on project P yesterday?A. 2B. 3C. 4D. 5E. 6Answer: E

138.A sum of $1000 was invested at the R percent semi-annual interest, compounded. If at the end of one year the total interest is $80.56, what is the possible value of R?A 5<R<6. B 6<R<7 C 7<R<8 D 8<R<9 E 9<R<10Because the interest income is slightly above 80.00, then pick 8%

Page 60: Mitul's 2007 GMAT Sep

first, semi-annual compound. You will find the interest income will be 81.6, $1.04 higher than 80.56. So the choice must be just slightly lower than 8%. So choose C.

139.40% members of a community are women. Out of this 30% of the women are 40 years or more. There are 1800 men. How many women are 40 years or more?

1800 is 60%. 40 % =  1200. out of this 30% = 360

140.We need to find the area of the figure. ABC=60 and ADC = 90 andDAB = DCB

141.What is the range of the numbers 2, 3, 4, 6, 7, 9, 10, 12, 20, and x?1) The mean is 92) x is the median

Answer :D

If the mean is 9, we can determine the number x.From II, if x is the median, we can determine x, from the given series.

hence, D.

142.PS: (x-y)^2+2*y^2=27, x=?Answer: 6

Did not understand the question. There are two unknowns.. so how to determine x?

143.(47.5/0.0475)^3=?Answer: 10^9

144.The average value of five numbers is 6, is the standard deviation greater than 10?1) Four of the numbers are 162) One of the numbers is ¨C4

From I, we can calculate the whole series..16,16,16,16,x. since the avg is known, 30 = 64 + x; x = -34Hence,we can determine the median of the series.

From II, i cannot make out anything..

Page 61: Mitul's 2007 GMAT Sep

Answer: A

145.One box contains 30 purple balls and 20 red balls, another box contains 15 purple balls and 35 red balls, if Wendy randomly chooses one box and draws one ball from it, what will be the possibility that the ball will be purple?The probability that eighter of the bags is picked is (1/2).Hence, the probablilty of picking the ball from 1st bag + probability of picking the boll from 2nd bag.

Answer: (1/2)*(30/50)+(1/2)*(15/50)=9/20

146.M, N and K are integers. Is the product of them divisible by 3?1) The sum of them is divisible by 32) M, N, and K are consecutive integers.

Answer: B1) 5, 8, 11

147.If An= A(n-3)+18, the first several numbers are 102,108,114, which of the following numbers is in the sequence?

the above is a progression, with d = 6 and a = 102

Answer: 594An=102+6n

148.If line m is perpendicular to line k, is line k intersects to line x=-1?1) The slope of line m is 32) Line k (or m?) passes through the point (0,4)We must visualize that x=-1 is a vertical line with slope of infinity.hence, any line with a defined slope will intersect the line.

From II, we cannot determine the slope of the line and hence, we can not confirm.

Answer: A

149.There are three kinds of bonus, each is $735, $150, $75. The company will give bonus to its employees. Each bonus has to be signed to at least one person. Now the company has the total bonus of $64,800. What is the minimum

Page 62: Mitul's 2007 GMAT Sep

number of employees who can get the bonus?

Since, min ppl have to get the profits, max profits need to be given out...

Hence, number of $735 = 88.Remaining is $120, which can be given to 1 person, hence, the total number of ppl to whom profits can be given is 89.

Something wrong with the Answers options.

A.10  B.11  C.12  D.13  E.14

150.If x is positive, is x<1?1) x<x^1/32) x^3<x

A positive number is smaller than itself when it is squared is when it is under 1.

From II, we can determine the same information.

Answer: D

151.It is same to Q104If d1, d2, d3, d4 are four different integers, whether is 0.d1d2+0.d3d4 greater than 1?1) The least number is 42) (0.d1d2)(0.d3d4)>0.5Answer: D1) Let them be 4, 5, 6, 7, then both 0.47+0.56 and 0.46+0.57 are greater than 12) Let them be a, b, c, dThen ab*cd/100*100>1/2ab*cd>5000ab+cd>/=2*(ab*cd)^1/2--->ab+cd>2*5000^1/2=100*2^1 /2 =141

0.ab+0.cd>1.41

152.The total radio and camera sold in a store is about N, how many radios was sold?1) The radio's mean price is N12) The camera's price is N2

From I, we cannot determine, eighter of the two numbers.. hence, not possible.

Same for II,Hence, E.

Page 63: Mitul's 2007 GMAT Sep

Discussions : MJJ 152-164

153.DS: X<5?1). |x-5|>02). x^2 + x <5

From I, we cannot determine if x<5. If X is 4 or 6, we still get an answer of 1.from II, it solves as x(x+1) <5, in both ways, x is eighter 0 or less than 4. Hence, B.

Answer: B

Discussions : MJJ 152-164

154.DS: x is a prime, x=?1) The sum of the factors of x is 242) The product of the factors of x is 23

From I, we know that x is prime, whose factors are itself and 1, hence, we can tell that the number is 23.from II, we can definetly tell that the prime number is 23.

hence, D.

Answer: DDiscussions : MJJ 152-164

155.Of a group of 200 people, how many are male and with brown hair?1) 50 percent of the people are brown hair2) 60 of the people are male

From I , we can determine only the total number of ppl with brown hair, which also contains women.from II, we can tell the number of males in the grp.

From both, we still cannot find out the number, since we do not know the number of women with brown hair.

Answer: EDiscussions : MJJ 152-164

156.If [X] is defined to be the number greater than x, y>-3£¿1) [y/2]=02) [y/4]=0Answer: E1) [y/2]=0 >>> 0>y/2        &nbs p;     

Page 64: Mitul's 2007 GMAT Sep

2) [y/4]=0 >>> 0>y/4

Discussions : MJJ 152-164

157.n+k=0?1) nk=02) n-k=0

from I, we cannot tell , if n or k is zero.From II, we can tell than n and K are same.

From Both, it is infered that, both n and k are zero.Hence, C.

Answer: CDiscussions : MJJ 152-164

158.A rectangle's dimension is x by y, where x<y. If the perimeter is x+16 and areas is 24, x=?Answer: x=4Perimeter = 2x + 2y = x + 16 => x + 2y = 16;Area= x*y = 24 => x = 24/y, => 24/y + 2y = 16, 24 + 2y^2 - 16y = 0, y^2 - 8y + 12 = 0 => (y - 6)(y -2) = 0 => y = 6, or y = 2, since x < y, then y has to be 6, thus x = 24/6 = 4

Discussions : MJJ 152-164

159.Line L passes through point (1,4), and the product of its intercepts with axis-x and axis-y is negative. Which of the following point is on the line L?A. (2,3)B. (5,7)C. (4,2)D. (4,-1)E. (5, -1)Answer:When u draw it out, the line has to go through either positive-x and negative y, or positive y and negative x, only (5, 7) makes this happen, thus it's 2)

Discussions : MJJ 152-164

160.If (0.00048*10^n)/19200<2*10^10, what is the greatest possible value of n?Answer: 17Rethink the problem as 0.0005 *

Page 65: Mitul's 2007 GMAT Sep

10^n / 20000 < 2*10^10 => .0005 * 10^n < 4*10^14 => 5* 10^(n-4) < 4 * 10^14, thus n - 4 = 13 => n = 17

Discussions : MJJ 152-164

161.Sequence a1, a2, a3 ... is such that, a3=a2+a1,a4=a1+a2+a3, ... If an=P, a(n+2)=?Answer: 4Pa(n+2)=a(n+1)+an+a(n-1)+...a(n+1)=an+a(n-1)+...an=P=a(n-1)+...Then, a(n+2)=4P

Discussions : MJJ 152-164

162.If one man, two children, one woman are to be selected from eight men, ten children, ten women, respectively, how many ways are possible?Answer: 3600C1,8*C2,10*C1,10=3600

Discussions : MJJ 152-164

163.5^21*4^11=2*10^n, n=?Answer:21

Discussions : MJJ 152-164

164.Is the least element of A less than that of B?1) The range of A is bigger than that of B2) The biggest element of A is smaller than that of BAnswer: CA2-A1>B2-B1, A2<B2, >>>A1<B11) A's range is bigger than B's range, but since A and B could be anything, it is not sufficient to establish the relationship between the smallest elements in each A and B, thus 1) is not sufficient2) The biggest element in A is smaller than the biggest element in B, since we know nothing of the range of the two sets, it's not sufficient either1) and 2), we know A's range > B's range, a(the biggest in A) < b(the biggest in B), since A's range = the biggest - the smallest, if A's range > B's range => the biggest in A - the smallest in A > the biggest in B - the smallest in B, since a < b

Page 66: Mitul's 2007 GMAT Sep

then the smallest in A has to be less than the smallest in B, thus 1) and 2) together are sufficientTherefore the answer is C

Discussions : MJJ 152-164

165.Of some five numbers, the average is 10, mode is 15, the greatest number is equal to  the mode, and the median is 12. What is the least number?1, 9, 13, ...Answer: 1

From the information, lets make a series...  No1, No2, 12,No3, 15.Since, least numer is asked, lets plugin ans check.Also, mode is 15, hence, 15 should occue atleast twice.

Let the series be 1, No2, 12,15, 15.

The sum is 43. For the avg to be 10, the sum should be 50.The second number is 7. Hence, the only possible least number is 1.

Discussions: MJJ 165-174

166.DS : x>y?1) x^2>y^32) x^3>y^4

From I, we cannot determine if x is negative or not, and still x^2 > y^3.Eg:- x = -4, y = 2, x<y, but x^2 = 16 > y^3..

Discussions: MJJ 165-174

From II, not sufficient, hence, not possible.

From I and II, still not possible.

Answer: E

167.In one hour, H can produce three dozen of something and T can produce four dozen of something. If they took 14 hours to produce 77 dozen of the product, what is the least simultaneous working hours? (It means that, during the 14 hours of period, T and H worked simultaneously for some time, then divided the other

Page 67: Mitul's 2007 GMAT Sep

time.)Answer: 7Let a to be the H's hour, b to be the T's time, c to be the common time.3a+4b+(3+4)c=77 and a+b+c=14=>3a+4(14-a-c)+7c=77 = =>3c=a+21When a =0, the c has the least time, 7

Discussions: MJJ 165-174

168. P---Q---------------R---SThe locations of four points in the number line is shown above. If PR=OS, PR=5PQ, P is 1/3, S is 1/2, what is R?Answer: 17/36pr = qs, rp = 5 * pqSince ps = pr + rs = pq + qs, and pr = qs => rs = pq, let rs = pq = x, then since pr = 5x, then ps = 5x + x = 6x = 1/2 - 1/3 = 1/6 => x = 1/36, thus r = s - 1/36 = 1/2 - 1/36 =17/36Discussions: MJJ 165-174

169.If An=200+0.2*A(n-1), and A1=200, A50 will fall in which of the following regions?(230,239),  (240,250), ...Answer: (240,250)A(n)=200+0.2*A(n-1),A(n-1)=200+0.2*A(n-2)=>A(n)-A(n-1)=0.2[A(n-1)-A(n-2)]=0.2^(n-1)(A2-A1)=0.2^(n-1)*40=> A50-A49=0.2^49*40 and A50=200+0.2*A49=>A50=200/0.8-0.2^49*40=250-10x(0.2^49)Therefore, 240<A50<250

Discussions: MJJ 165-174

170.Someone invested a certain amount of money at the 2.5 percent annual interest rate. At the end of the fifth year, the total amount in the account is $11250. How much did he invest?The number maybe inaccurate, pay attention on the thinkingx*(1+2.5%)^5=11250x+12.5%*x+10*2.5%^2x+...(can be neglected)=11250Above all: x*(1+2.5%)^5 approximately equal to x+12.5%*x

Discussions: MJJ 165-174

Page 68: Mitul's 2007 GMAT Sep

171.About a certain kind of records of a whether station. Which of the least values in July and in August is bigger?1) The range of the records in July is greater than that in August.2) The greatest record in July is less than that in August

From I, we cannot derive information on which has a  greater value.From II, we can determine that the greatest record is in August.Also, But, does not tell which month has the greater lowest value.From I and II, it is clear that August has the greater least value, since the range of records in July is more than in August and August values are higher.

Answer: C

Discussions: MJJ 165-174

172.A circle, which circumference is 1 mile, is equally divided by nine points O, P, Q, R, S, T, U, V, and W. At the speed of 15mile/hour, someone started out from the point O. After three minutes, which region did he arrive in?

Since , the circumference is 1 mile, each section is 1/10 mile. distance travelled for 3 min at 15 mile/hr is 3* 15/60 = 3/4.3/4 = 7.5/10, hence, in the 7th section, which is UV.

Answer: UV

Discussions: MJJ 165-174173.9^x + 9^(-x) = 62, 3^x + 3^(-x) =?Answer: 8[3^x + 3^(-x)]^2=9^x + 9^(-x)+2=62+2=64 >>> 3^x + 3^(-x) =8

174.One of the angles of a pentagon is 80 degree. If the other four angles have the equal measure, what's it?Answer: (n-2)*180-80/4, n = 5 , 540 - 80 / 4, ans = 115.

Discussions: MJJ 165-174

175.PS: invest 200,000, interest

Page 69: Mitul's 2007 GMAT Sep

5%, annual compounding, then after one year invest  this investment and the interest earned to a two yrs program, interest is y%, compounded annually, after two yrs, the interest is 8484, ask what is y?A. 2.0, B2.5 C. 3.0 D.3.5 E4.0

P = 200,000; A =  p(1.05) = 200,000 +10000 = 210,000 .210000(1+y/100)^2 = 8484By plugin method, A is the answer, y = 2%.

Answer: A

Discussions:MJJ 175-182

176.DS: x and y are integers, and when x+y divided by 11, the remainder is r, ask for r:1) When x-2 is divided by 11, the remainder is ...2) When 35-y is divided by 11, the remainder is 1 (I am not sure)Answer is C.

Discussions:MJJ 175-182

177.There are 6 students, and they are chosen to form 3 groups to make presentation. Each group will be assigned Asia, Africa or America as their presentation subject. And the question is how many groups could be formed.The first 2 ppl can be chosen in 6C2 ways. the next 2 in 4C2, and other 2 in 1 way.

Answer: C(2, 6)*C(2,4)*C(2,2)=90.

Discussions:MJJ 175-182

178.If 0.02<X- root10<0.021, the value of (1/root10 ¨C1/x) is most close to which of the following numbers?A.0.2   B.0.02   C.0.002   D.0.0002   E.0.00002   Answer: C1/root10 ¨C1/x=(x-root10)/x*root10Because x approximates root 10, therefore (x-root10)/x*root10 approximates (x-root10)/10= => 0.002

Page 70: Mitul's 2007 GMAT Sep

Discussions:MJJ 175-182

179.X= 15 (0.0001) (0.00001) = a (10)^b; 0.1<a<1, b =?

The main idea is to equal the number of decimal places in the LHS.Answer: -715 (0.0001) (0.00001)=0.15x(10^-7)=a (10)^b

Discussions:MJJ 175-182

180.DS: S0 =0, S10 =?1) Si = i + Si-12) Si + Si-1 = i^2

From I, we can perform iteration and find the value of S10, s10 = 10 + S9;s9= 9 + S8.. and so onFrom II, same as I. We can calculate the same.

In GMAT, for such question, we need to see if this pattern can be traced back to the given value!!

Discussions:MJJ 175-182Answer: D

181.P is an integer, is P a prime number?1) P+3 is a prime number2) P^2+3 is a prime number

From I, we can determine that, if P+3 is a prime number, P cannot be a Prime Number.Sine, to be a Prime number, the number should be an odd number.But, we have an exception,.ie, 2. Hence, Not possible.

From II, P^2 +3 is a prime number, P^2 is an even number, hence, P cannot be prime. Again, 2 is an exception. Hence, Not possible.

From I and II, the only possibility is 2. Hence, Possible.

Answer: C

PS: For all Prime Number

calculation, pluggin 2 first..

Discussions:MJJ 175-182

Page 71: Mitul's 2007 GMAT Sep

182.DS: x^2+bx+c=0, r and s are the solutions of the formula. Is r*s<0?1) b<02) c<0Answer: BX^2 + BX + C = 0, (X-R)(X-S)=X^2 + BX + C=0Then B= - (R+S), C=RSFrom statement 2, we know that c<0, R*S<0

Discussions:MJJ 175-182

183.If x is one of the a group of 21 numbers and is four times the average of other 20 numbers, x is what fraction of the sum of all the numbers?

If X is one of the 21 numbers, and is equal to 4 times the average of other 20 numbers, then

x = 4A, where A = avg of the remaining 20 numbers.

Sum of all numbers is x + 20A= 24A; Hence, the 4a/24a = 1/6

Answer: 1/6

Discussion: MJJ183-188

184.The line represented by which of the following equations is perpendicular to the line represented by y=5x?y=-5x, y=-x/5, y=x/5,...

The product of 2 perpendicular lines should be -1. the slope of the given line is 5. So we need to select a line with slope of -1/5.

Which is y=-x/5.Discussion: MJJ183-188

185.Three of the 15 books are defect. If two books are to be selected from the book, what is the probability that both two books are not defect?

Number of ways you can pick 2 books from 15 = 15C2.Number of ways to pick books without defect = 12C2. Probability = 12C2/15C2.

Page 72: Mitul's 2007 GMAT Sep

Discussion: MJJ183-188

186.If xy=165, and x>y, the least possible value of x-y is:2, 4, 8, 23, 15

After factorizing 165, we get 11 and 15, hence, the least difference is 4.Discussion: MJJ183-188

187.In a certain class, the range of boy's heights is 14 inches and the range of the girl's heights is 12 inches. What is the range of the heights of all the students?1) The shortest boy is 3 inches shorter than the tallest girl2) The height of the tallest boy is 74 inches

From I, we can determine the range which is 14 + 12 - 3.

From II, we cannot determine the girls height from this, hence, we cannot.

Answer is A.Discussion: MJJ183-188

188.A person had two accounts. He invested $1000 in an account paying 4% interest annually and $2000 in another account paying 2% interest annually. Another person had only one account to which he invested $3000. If they invested at the same time, how much should the second person's rate be, so that in the end of one year, they earn the same amount of interest?

person A, interest = 1000*4/100 + 2000*2/100 = 40 + 40 = 80.person B, 80 = 3000*r/100; r = 8/3 %

Discussion: MJJ183-188

189.Set A contains five negative numbers. Set B contains six numbers, one of which is 0 and three of which are positive. If one number is selected from A and another is selected from B, what is the probability that the product of

Page 73: Mitul's 2007 GMAT Sep

them is negative?

From set B, the probability that a -ve number is always picked from A is 1.from set A, the probability that a +ve number is picked other than 0 is 3/6 = 1/2.

Probability = 1/2.

Answer: 1/2

Discussion:MJJ 189-199

190.6^(a+4)/[2^(a+2) * 3^(a-2)]

   (2^a *3^a * 6^4)/(2^a*3^a * 2^2*3^(-2) )   =6^4*9/4

Discussion:MJJ 189-199

191.The sum of 1/41, 1/42, 1/43, and 1/44 is between:Answer: 4/41~4/44

Discussion:MJJ 189-199

192.A person plans to decorate his house. Company A's service price is $2.4 per square meter. Company B's service price is $3 for each of the first 500 square meters and $2 for each square meter there after. Which company's service is more economical?1) The total area of the house is greater than 1000 square meters2) The total area of the house is less than 1200 square meters

from I, we cannot determine the actual price, since, no upper limit is mentioned.From II, whatever price is considered within 1200, we get consistent values....

hence, B.

Discussion:MJJ 189-199

193.If n is a positive integer, is n*(n+1)*(n+2) divisible by 12? [or, is 12 divisible by n*(n+1)*(n+2)?]1) n*(n+2) is even2) n+1 is even

From the question, we know that

Page 74: Mitul's 2007 GMAT Sep

the series is always divisible by 3. So, to prove that 12 is either divisible or vice versa..we need to know if there is an even number in it.

From I, yes, we can determine.From II, yes, we can determine.

The author's answer is DThe question maybe is incomplete.

Discussion:MJJ 189-199

194.A person loaned $2000 from a bank for two years. In the end of every six months, he paid 5% interest first, and then paid $500 back to the bank. When he paid off all the money he loaned, how much interest had he paid?Answer:2000*5%+1500*5%+1000*5%+500*5%

Discussion:MJJ 189-199

195.DS. x/y < b/c ?1). cx-yb<02). bx-cy<0

From I, we can determine the equation by rearranging it.cx<yb; x/y < b/cFrom II, we cannot dermine.

Hence, A

Discussion:MJJ 189-199

196.PS. x is a positive integer. If both the units' digit of (x+1)^2 and the units' digit of (x+3)^2 are 1, what is the units' digit of x-3?X = 8.Answer: 5

Discussion:MJJ 189-199

197.The formula for Celsius degree exchanging to Fahrenheit degree is F=9/5X+32. If five data measured with Celsius way are A, B, C, D, and E, where A<B<C<C<E, when they are exchanged to Fahrenheit degree, the range will be:MY ANS: 9/5(E-A)

Discussion:MJJ 189-199

Page 75: Mitul's 2007 GMAT Sep

198.If two of the sides of an isosceles triangle are 6 and 6*root 3 long, respectively, what is the perimeter of the triangle?1) One of the angles is greater than 90 degree2) The perimeter is less than 24

From I, if one of the angeles is greater than 90, the sides along which the angle is based has to be short, which is 6.hence, the perimeter is 6+6root3

from II, it is said that the perimeter is less than 24. The only other option is 6+6+6*root3

Discussion:MJJ 189-199Answer: D

199.Of seven consecutive integers, how many are multiple of 6?1) The median is the multiple of 62) The sum of the numbers is the multiple of 6

From I, we can tell that 3 numbers on eighter side of the number series is not a multiple of 6.From II, by plug-in method we can determine.

Answer: DDiscussion:MJJ 189-199

Questions that appeared in my GMAT today(08/18):

200. Six students form 3 pairs and they have to be assigned to 3 different continents. How many ways can they be assigned.

Answer:  6C2 * 3! = 90 (Discussed earlier).

201. If r*s = s*r, which of the following are correct.

I) r+s-rsII) r/s + s/rIII) cant remember. but III is not the rite choice.

I&II are the correct.

202. Given the coordinates of a triangle in xy plane. AB parallel to

Page 76: Mitul's 2007 GMAT Sep

x axis and A coordinates (a,b), B coordinates (6a,b), C coordinates (c,4b)

A) a = 3/4bB) Area of the triangle is 90 units.

I have selected E but I guess its C.

203. Out of X students, 64% are educated, 48% are educated males. What is the fraction of female that are educated.

Ans:  16/64 = 25%.

204. An amount 150K invested in 1984 grows X% in 1985 and Y% of 1985 in 1986. What is the difference b/w amounts in 1986 and 1984.A)  X + Y = blabla....B) (1+x/100)(1+y/100) = given amount.

Ans: B.

205.A sales person wants to start five cars with their keys, but he doesn't know the right keys to them. If he starts the five cars with the five keys randomly, which of the following cannot be the number of the cars that he exactly starts?A.0B.2C.3D.4E.5

If you are able to start 4 cars, then automatically, the 5th will start, since the only other key left, will start the last car.Hence, the answer is 4.

Answer: 4

Discussions: MJJ 205-215

206.3x-2y =?1) x/0.2 - y/0.3 = 9 2) x/0.3 - y/0.2 = 7

From I, we can derive the same equation that is asked for.From II, it is not possible to derive

Page 77: Mitul's 2007 GMAT Sep

the same equation.

Hence, A.Discussions: MJJ 205-215

207.If 15% of the plants have fungus and die, and 40% of plants with fungus did not die. What percent of plants has fungus?

Confusing... Not clear...Discussions: MJJ 205-215

208.There are five pairs of shoes with different colors in a box. If two shoes are to be selected from the box without replacement, what is the probability that they have the same color?

The number of ways that you can pick 2 shoes from 10 is 10C2.The number of ways that you can pick 2 shoes with same color is 5 ( 5 pairs)

probability is 5/10C2

Answer: 5/C(10,2) = 1/9

Discussions: MJJ 205-215

209.If [(2X)^(1/2)]*[(3W)^(1/4)] = [(2Y)^(1/2)]*[(3Z)^(1/4)], and X=4Y, W is what fraction of Z?Answer: 1/16Discussions: MJJ 205-215

210.In a certain store, the original sale price for something is X% of the retail price. One month later, the discount sale price is Y% of the original sale price. If the retail price is 72, what is the discount sale price? 1) X*Y = 5600 2) 8X = 7Y 

From I, we can calculate the discount sale.the sale price is retail price *x. Now, the discount price is retailPrice*x*y.Retail price is given and product is given.

From II, we can only get teh ration of the %. Not usefull

Page 78: Mitul's 2007 GMAT Sep

Hence, A.

My answer: ADiscussions: MJJ 205-215

211.If x is an even positive integer, which of the following is (are) the remainder of (3x)/5?I.0, II.1, III.2

ex, 15,21,12Answer: all are right

Discussions: MJJ 205-215

212.If C(3,5)=C(x,5), x=? (x is not 3)Need not calulate... C(r,n) = C( n-r,n)

Answer: x=2

Discussions: MJJ 205-215213.If x*(2^y)=176 and x is a prime number, x*y=?

From the above , we notice that 176 is a factor of 11. The remaining is 4. Hence, the product 44.

Answer: 44

Discussions: MJJ 205-215

214.In a xy-plane, a line passes through point (0,4) and (4,0), which equation represents the line?

My Answer: x/4 + y/4 = 1Discussions: MJJ 205-215

215.The length of a certain nail differs from the average length of the entire lot of nails in amount between d and 2d (note: either shorter or longer, the difference is between d and 2d) ask if the length of this certain nail is > 751) d= 42) Mean length = 80

From I, we cannot determine anything.From II, we still cannot determine anything.

From I and II, the range is between 76 and 72. Still cannot determine.

Page 79: Mitul's 2007 GMAT Sep

Answer: EDiscussions: MJJ 205-215

216.Paul has x dollars. Tom's money equals Paul's plus twice the Anita's. If Paul and Anita cost one dollar, respectively, Paul's money will twice Anita's. How much money, in x, does Tom have?Answer: T=2X+1Another version:Paul has m dollars. Tom's money is twice the sum of Paul's and Anita's. If Paul and Anita cost three dollar, respectively, Paul's money will twice Anita's. How much money, in m, does Tom have?Answer: 3m+3

217.Bruce bought only $0.32 stamps and $0.23 stamps. How many $0.23 stamps did he buy?1) He bought $0.87 worth of stamps.2) ...

If total number of stamps or the relative difference between the number of stamps is given, its C.Else, E.

218.Is x^2-y an integer?1) Both x and y are integers2) x*y is negative

From I, yes we can determine if the difference is -ve, since, both are integers, the difference will also be negative.

From II, we do not know if eighter is not an integer.

My Answer : A

GivenAnswer: E

219.If 140*n is the square of an integer, where n is an integer. What is the least value of n?

140 = 10 * 14; 2 *5 *7*5. Hence, to make it a complete square , we need 35.Answer: 35

220.From 1990 to 1993, the sales value increased by 40%. From 1990 to 1995, the sales value increased by 40%. What was the percent increase from 1993 to 1995?Answer: 29%

Page 80: Mitul's 2007 GMAT Sep

I am not getting the same answer..

221.A contains two red flowers and x white flowers; B contains five red flowers and three white flowers. X=?1) If one flower is selected from A and B, respectively, the probability that both two flowers are red is 1/42) If one flower is selected from A and B, respectively, the probability that both two flowers are white is 9/40

From I, yes, we can determine the number of white flowers.

the prob is (1/2)*[ x/x+2 + 5/8 ] = 1/4.

Similarly, From II, we can find out the number of white flowers......

hence, D

222.If 10^x*(0.0099999)>10^3, x at least is: 6

223.If the symbol * represents either addition, subtraction, multiplication, or division, what is the value of 6 * 2?1). 10 * 5 = 22). 4 * 2 = 2

From I, we can determine that * represents Division.From II, we cannot determine whether * is subtraction or division.Hence, A.

Discussions: MJJ 223-235

224.In town X, 64% of the population are employed, and 58% of the population are employed male. What is the percentage of the female that are employed?A .16% ...D.32% ...

Answer : Humanly impossible !!

Discussions: MJJ 223-235

225.If r@s=s@r  which of the following can define the equation? 

1. r@s=r+s-rs  2. r@s=(r+s)(r-s)3. r@s =r/s+ s/r

Page 81: Mitul's 2007 GMAT Sep

Its 1 and 32 is ruled out. If eighter is negative (r-s) part of it will eighter be -ve or positive, which will not yield the same result.

eg, r=2,s=-3 r@s = (-1)(5) = -5 s@r = (-1)(-5) = 5. Hence, 2 is ruled out,

Discussions: MJJ 223-235

226.What is the equation for a line which passes (30,0) and (0,20)?

(y - 20)/x = 0-20/30 = -2/3 ; -2x = 3y - 60; 2x + 3y = 60

Discussions: MJJ 223-235

227.What is the average of 59 consecutive integers?1) The sum of the largest number and the least number is 12) The sum of all is 1

From I, we know that the sum of largest and lowest number will be negative only if either is negative but not both.Here, the largest being 29 and the lowest being -28. The sum is 1. So , we know the series and hence, can find the average.

From II, if we know the sum of all the numbers, the avg will be 1/59. Hence, enuf.

Hence, D.Answer: DDiscussions: MJJ 223-235

228.The sales price of a picture is 10% more than the cost to it. In a promotion, the discounted price is 10% lees than the original price. What is the relation between the discounted price and the original price?

Let us consider the price of the item as 100.Sales price = 110the discounted price is 10% lees than the original priceDiscounted price = 90

Page 82: Mitul's 2007 GMAT Sep

hence, the relation is 10/100 = 10% less

If the discounted price is 10% lees than the sale priceDiscounted Price = 110 -11 = 99.

Then, the relation is 1% less

Given Answer: discounted price is 1% less (not more) than the original price.

Discussions: MJJ 223-235

229.DS. (x+y)^2=?1) x=y-32) x and y are prime numbers.

Only the difference of 5 and 2 is 3Answer: C

Discussions: MJJ 223-235

230.What is the relationship between km/h and m/second?1 KM/HR = 5/18 m/sec

Discussions: MJJ 223-235

231.Someone invested $20,000 at x percent annual interest and $25,000 at y percent annual interest. At the end of the first year, how much interest did he earn from the $20000 investment?1) The interest from $20,000 is $300 more than that from $25,0002) x=(2/3) yAnswer: C

From I, we can only get the relation between the two folrmulas. We will still have two vars X and Y.From II, we will get the relation between the two interest rates, still the interests earned by the two is not known.

From Both, we can realate the above unknowns and get a solution.hence, C

Discussions: MJJ 223-235

232.Of the members in a certain group, 62% are shareholders and 47% are employees. How much percent of the members are shareholders but not employees?

Page 83: Mitul's 2007 GMAT Sep

38%, 45%, 53%...

100=62+47-(both)both = 9%So Only S = 62-9=53%

Discussions: MJJ 223-235

233.A circle is divided to six sectors equally, and another circle is divided to eight sectors equally. What is the ratio of the measure degrees of their central angles?

 6 divisions: angle of each sector is 360/6 = 60 deg 8 div: angle of each sector is 360/8 = 45 deg  ration of 6 sec/8 sec = 4/3

Discussions: MJJ 223-235

234.(x^1/n*y^1/n)^n=?

=> (xy)^1/n*n = xyAns:xy

Discussions: MJJ 223-235

235.Three machines have the identical rate. If two of them work together, it takes k hours to complete a certain work. If all of them work together, it takes two hours less to complete the same work. K=?

Let the rate of each machine be x. Rate at which each machine works in one hour is 1/x.

In K hours, they work k/x.

Two machines, will work at 2k/x.

With three machines, its 3(k-2)/x. Since the work done is same,

3(k-2)/x = 2k/x; 3k -2k = 6; k=6.

Discussions: MJJ 223-235

236.A plant can be green or yellow, and it leaf can grow to be curve or straight. If the probability that grow to be green is x, to be straight leaf is 1/4, and to be green and curve leaf is 3/8, what's x?

Page 84: Mitul's 2007 GMAT Sep

Probability to grow straight leaf = 1/4Probability to grow curly leaf = 3/4

Probability to grow green and curly = x*3/4 = 3/8;x = 1/2

Discussion: MJJ 236-243

237.DS: m and n are integers, is m^n an integer?1) n^m is positive2) n^m is an integer

if m is negative in eighter case, we get a fraction. Hence, not integers.Thus , cannot be solved.

Answer: E

Discussion: MJJ 236-243

238.If N is an even number and E is an odd number, which of the following cannot be an integer?A. N/E B. E/N C. N/(E+1) D. (N-1)/E E. (N+2)/(E-1)

Ans: B. You cannot divide an ODD number by even number

Discussion: MJJ 236-243

239.In a company conference, each female employee carries three pens and each male employee carries two pens. How many female employees attend the conference?1) The total number of the pens is 262) The total number of the employees is ...

From I, 3x + 2y = 26. Not possible to solveFrom II, x + y = Z.. not possible

From I and II, we can determine the values of X and Y.

Answer: C

Discussion: MJJ 236-243

240.A simple but tricky question:

Page 85: Mitul's 2007 GMAT Sep

C5,12=12*11*2*3*Answer:B:8*9*11*

Discussion: MJJ 236-243

241.If x>0, is y>0?1). x>7/2y2). x>|y|

From I, we cannot determine the value of yFrom II, Not possible to detect, becuse either way x may be greater than Y. i.e, if y were -2 or 2, x is greter than both values.Answer : E

Given Answer:I am not sure about statement 2, chose E

Discussion: MJJ 236-243

242.Three machines in a coalmine are used for transporting coals. X can transport 1/4 of a certain amount of coal in 4 hours, Y transport 1/2 in six hours, and Z transport 1/3 in eight hours. If they work together for 24 hours, how much will they transport?

X can transport in 1 hr : 1/16Y in : 1/12 and Z in:1/24total work in 1 r = 1/16 + 1/12 + 1/24In 24 hr = 3/2 + 2 + 1 = 9/2 = 4.5

Discussion: MJJ 236-243

243.The average of the data in 12 weeks in a certain company is n. If 24, the data in a certain week, is replaced by a number, the average became n-3/4. What is the number?15,18,23,24,33Answer: 15

12n - 24 + X/12 = n-3/4n - 2 + X/12 = n -3/4; X/12 = 5/4; X = 15

Discussion: MJJ 236-243

244.If the slope of line represented by ax-by=c is 2/3, what is the value of b?1) a = 4

Page 86: Mitul's 2007 GMAT Sep

2) c=...

The slope is given by -a/b.

From I, we can determine b, since slope and a is given.

From II,we cannot determine the value of b.Answer: A

Discussions: MJJ 244 - 250

245.Working together at the identical rates, three people can complete a certain work in 36 hours. If another additional person, at the same rate, working together with them, how many hours will be decreased to complete the work?

Let x be the work that is done by each person, then3/x = 1/36; x = 108 hrs.4/108 = 1/27; 27 hours.The difference is 9.

Answer: 9Discussions: MJJ 244 - 250

246.4^x+2^2x=2^200, x=?Answer: x=199/2

247.DS: Which of 1/x, x, x^2 has the least value?1) x>02) x<1

From I alone, we cannot determine the OA.

from II, we do not know in which range does x lie.. 0 to 1 or 0 to -infinity.

From I and II, we know the range in which x lies, and hence, can determine.Answer: CDiscussions: MJJ 244 - 250

248.A store sells cake mix, muffin mix and bread mix. Of the 100 customers purchased mix of the store, 50 purchased at least cake mix, 40 purchased at least muffin mix, 20 purchased both cake mix

Page 87: Mitul's 2007 GMAT Sep

and muffin mix. How many customers purchased only bread mix?

the total number if people = 100Total number of people who bought cake mix = 50Total number of people who bought muffins = 40Both = 20

so, 100 = 50 + 40 -20 + only bread;bread = 30

Answer: 100-¨50+40-20£©=30Discussions: MJJ 244 - 250

249.A group cruise contains n people, n is between 50 and 150. What is the total cost of the group?1) The total cost of the group is (300n-n^2) dollars2) Each person spent $160.My answer: C

From I or II, we cannot determine the numbe of persons,

From I and II, we determine the equation 300n-n^2=160n, from which we can determine n.

Hence, C.Discussions: MJJ 244 - 250

250.If x, y, w, z are integer and x+y=w+z, which of the following about which of the numbers is (are) even is right?a) None, ...d) None, two, four  e) None, one, two, three, four

Incomplete QuestionDiscussions: MJJ 244 - 250

 Given Ans: D

Edited by Spiderman on 04 September 2005 at 4:47am

Back to Top  

meher13VIP

Posted: 04 August 2005 at 2:43pm | IP Logged

Page 88: Mitul's 2007 GMAT Sep

Joined: 14 October 2004India

Posts: 27Gender: Not Specified

21. Ans is 128 and not 120. Convert the hrs to mins and back to hrs to ease the fraction

22. I agree with the point for E. But  and also will add that  it will be E still because we do not know the extent of AB<DE and  BC>EF. Hence, cannot say for sure the conclusion

23. E . Go by the basic counting pronciple to ease the confusion of P or C

24. Yes 8^6 =256K

25. Ans 4

26. Ans is 42

27.  6 a*a is the formula

28. 76.8  

Sol: SP-CP/CP= 0.2 and 80-CP/CP = 0.25 solving both equations we get SP= 76.8 ANS

29. E.

1)  and 2 ) Obviously not sufficient

Combining:  We have a st line from 3q and 4Q. This can go from either 2Q ( -ve slope)  or 1Q ( + slope).  Hence, answer is E.

Edited by Stone on 06 August 2005 at 9:01am

Back to Top  

sandilyaVIP~

Joined: 21 December 2004India

Posts: 68Gender: Male

Posted: 05 August 2005 at 2:09pm | IP Logged

spiderman,

31.If x and n are positive integers (or integers?), and when (n+1)(n-1) is divided by 24, the quotient is x and the remainder is r. r=?1) 2 is not the factor of n2) 3 is not the factor of n Answer: C

Can u please explain Q31.

I thought it was E

Back to Top  

Page 89: Mitul's 2007 GMAT Sep

SpidermanGMAT Tutor

Joined: 02 May 2005India

Posts: 945Gender: Not Specified

Posted: 05 August 2005 at 3:10pm | IP Logged

Hello sandilya,

The best way, is to plugin....

Take n =5, not divisible by 2 or 3.

(5-1)(5+1) = 2424/24 = remainder= 0

Take n=7,

(7-1)(7+1) = 4848/24, remainder = 0.

Hence, C

Even in the answer explanation, we take 6k +5, why? because.. 6k+5 is not divisible by 6. or by eighter 2 or 3.

cheers,

Back to Top  

gatgatgamVIP~

Joined: 11 April 2005Johnston Atoll

Posts: 477Gender: Not Specified

Posted: 05 August 2005 at 4:08pm | IP Logged

45.

r s tu v w x y z

Each of the letters in the table above represents one of the numbers 1, 2, or 3, and each of these numbers occurs exactly once in each row and exactly once in each column. What is the value of r?1) v + z = 62) s + t + u + x = 6

Statment I -- Something is wrong....

Statement II - Possible, since x and u, and s and t, cannot take the same value and they have to be 1 or 2. r =3

Answer =B

Given Answer: D

-------

Stat (1) v + z =6 is possible only if v=z=3

Page 90: Mitul's 2007 GMAT Sep

so we get the pattern

Row 1       r s tRow 2       u 3 w Row 3       x y 3

Row 2 and Row 3 already have a 3, so third 3 will be in Row 1.

Column 1      Column 2       Column 3

    r           ;          s           ;         t

    u           ;          3           ;        w

    x           ;          y           ;         3

 

Column 2 and Column 3 already have a '3', so third '3' has to occur in Column 1.

Combining above, third '3' has to occur in Row 1, Column 1, or (1,1) in matrix terminology.

Hence Stat (1) is also sufficient.

Thus the given answer (D) is right.

Back to Top  

FearFactorManager

Joined: 15 June 2005United States

Posts: 51Gender: Not Specified

Posted: 05 August 2005 at 5:02pm | IP Logged

42.On the first day, a company sells orange beverage mixed with the same amount of orange juice and water. On the next day, the company adds the water twice as much as the amount of orange juice sold on the first day. If the company sells a glass of the beverage at $0.6 on the first day, how much is the price on the next day if  the company get the same revenue?I choose $0.4

Let's say the first day company mixed, X amt of orange juice with X amt of Water to make 2X amt of orange beverage

Total revenue on the first day, 2X*0.6

Second day, company adds the water twice as much as the amount of orange juice sold on the first day...so total juice second day

 2X + 2X = 4X, Let the price be Y,

Page 91: Mitul's 2007 GMAT Sep

4X * Y = 2X * 0.6

Y = 0.3

Now if statement means, twice the amount of Orange beverage sold then total

 =  2X + 4X = 6X

so 6X*Y = 2X*0.6, Y = 0.2

I don't get 0.4 at all...unless i'm misunderstanding the Qn.

Back to Top  

gp1974700 Club

Joined: 17 January 2005United States

Posts: 413Gender: Not Specified

Posted: 05 August 2005 at 10:38pm | IP Logged

Yes, I do get the answer for Question 42 as 0.3. The given answer might be wrong.

Back to Top  

SpidermanGMAT Tutor

Joined: 02 May 2005India

Posts: 945Gender: Not Specified

Posted: 06 August 2005 at 7:35am | IP Logged

42. Its orange juice that is added twice and not the beverage..

so, it 3x * y = 2x * .6

y = 0.4

cheers,

Back to Top  

phoenixVIP~

Joined: 13 February 2005

Posted: 06 August 2005 at 9:02am | IP Logged

44. For (b^2 - 4ac)  to be greater than 0, dont we also need to know whether (b^2 > 4ac). Since the choices dont mention b at all, thus the answer is E.Other opinions?

Page 92: Mitul's 2007 GMAT Sep

IndiaPosts: 330Gender: Not Specified

Back to Top  

carnotVIP~

Joined: 01 August 2005Argentina

Posts: 39Gender: Male

Posted: 10 August 2005 at 11:53am | IP Logged

hello spiderman,

I am not OK with 75, why did you convert to meters instead of just converting miles per hour into feet per hour or feet per second.

30 miles per Hour = 150 feet per hour to cover 44 feet, it takes 44/150 of an hour so it will take 44/150 of 3600 seconds, which is 1056 seconds

Back to Top  

carnotVIP~

Joined: 01 August 2005Argentina

Posts: 39Gender: Male

Posted: 10 August 2005 at 11:57am | IP Logged

OK spiderman, i found conversion tables on the net and understood that it will take approx 1 sec to cover 44 feet at that speed, but didnt understand why i didn't get that result by doing what I did

thx anyways

Back to Top  

vcbabuVIP

Joined: 11 September 2004India

Posts: 3078Gender: Not Specified

Posted: 12 August 2005 at 1:42am | IP Logged

80.1/2+1/4+1/8+...+1/512=? A.0<S<1/2 B.1/2<S<1 C.1<S<3/2 D. 3/2<S<2 E. 2<S<5/2

This is geometric progression.

a= 1/2, r= 1/2 ,l = 1/512

Sum = (1/2 - 1/512)/(1/2) = 255*2/512 = 255/256 < 1.

<STRoNG style="FONT-WEIGHT: bold">(B)

sn = a ( 1- r^n) /( 1-r ) is ths gp summation formula .

how n is find out ,\.

Can so explain .

Page 93: Mitul's 2007 GMAT Sep

----------------------------------

pls xplain q 79

tons of thna x

Back to Top  

SpidermanGMAT Tutor

Joined: 02 May 2005India

Posts: 945Gender: Not Specified

Posted: 12 August 2005 at 4:17am | IP Logged

Hello babu,

Look at the same formula,

the numerator says.. a(1 - r^n)..

what is a*r^n = the last number in the sequence !!

Hence, the numerator becomes, (a-l), where l is the last number in the sequence.

I hope you have understood this concept.

cheers

Back to Top  

david21VIP~

Joined: 27 June 2005Monaco

Posts: 21Gender: Not Specified

Posted: 12 August 2005 at 2:17pm | IP Logged

69.A cuboid is 25 by 10 by 5, except the bottom, all the side faces are covered with paper. (the bottom maybe 10 by 5?) How much paper will be used?

Since, we are talking about wrapping the box, we have to calculate the surface area of the box except the bottom.

10*5 + 2(25*10+25*5) = 425

I think it is 800(250+125=375;375*2=750;750+50=800)

,but anyway you did a good job thank you

Back to Top  

iyer_sriniVIP~

Joined: 10 May 2005United States

Posted: 15 August 2005 at 6:04am | IP Logged

Spiderman: Can you explain 99 again. Sorry didnt get the question. Thanks for your time.

Page 94: Mitul's 2007 GMAT Sep

Posts: 23Gender: Not Specified

Back to Top  

saurya_s700 Club

Joined: 19 September 2004United Kingdom

Posts: 250Gender: Not Specified

Posted: 15 August 2005 at 6:11am | IP Logged

can somebody explain 104. I think it is A

Back to Top  

vishal.b21VIP~

Joined: 12 July 2005India

Posts: 511Gender: Not Specified

Posted: 15 August 2005 at 9:04am | IP Logged

42 : is 0.4

 

Back to Top  

vcbabuVIP

Joined: 11 September 2004India

Posts: 3078Gender: Not Specified

Posted: 15 August 2005 at 10:26am | IP Logged

 

16.A (n) = A (n-3) +18, and the first several numbers are 102, 108, 114 ... which of the following numbers are in the sequence?Reference key: An=96+6n

19.A person travelled from X to Y and arrived at Z. If the average speed in the whole trip is 15feet/second, what is the average speed during the route from Y to Z?1) The average speed from X to Y is 10 feet per second2) It took 20 seconds from Y to Z My answer: E

CAn so explain these q pls !!!!!!!!!!

43.X is hundredth digit of 0.0X. Y is thousandth digit of 0.00Y. What is the proximate maximum ratio of the two numbers?Reference key: If ask x:y, the answer is 9; if ask for 0.0x/ 0.00y, the answer is

Page 95: Mitul's 2007 GMAT Sep

90

44.In xy-plane, Y=ax^2+bx+c,does the graph intersect with X axis?1). a>02). c<0Answer: Cb^2-4ac>0

My answer is C.

47.Ds m and n are integers, is m^n an integer?1) n^m is positive2) n^m is an integerAnswer: E

My answer is E

Back to Top  

SpidermanGMAT Tutor

Joined: 02 May 2005India

Posts: 945Gender: Not Specified

Posted: 15 August 2005 at 3:54pm | IP Logged

Quote: iyer_srini Spiderman: Can you explain 99 again. Sorry didnt get the question. Thanks for your time.

The question is not mentioned properly.

I think what it's trying to say is: out of all trees planted WHICH DID NOT FLOWERED, trees in F which did not flower count as 15% OF THOSE; and 40% of F trees flowered, so what % is the number of trees planted in F overall?

cheers

Back to Top  

adityakhannaVIP~

Joined: 28 March 2005India

Posts: 10Gender: Male

Posted: 18 August 2005 at 9:32am | IP Logged

hi,

can someone pls explain q 18. its answer is mentioned as c, but i think it shd be e- since even after combining the answer, one doesnt know how many of the female students succeed?

18.How many of the students applying for making a speech succeed? 1) 2/3 of the male students and 1/3 of the female succeed.2) 26 male students succeed.Answer: C

Back to Top  

Page 96: Mitul's 2007 GMAT Sep

SpidermanGMAT Tutor

Joined: 02 May 2005India

Posts: 945Gender: Not Specified

Posted: 18 August 2005 at 12:06pm | IP Logged

Quote: adityakhanna

hi,

can someone pls explain q 18. its answer is mentioned as c, but i think it shd be e- since even after combining the answer, one doesnt know how many of the female students succeed?

18.How many of the students applying for making a speech succeed? 1) 2/3 of the male students and 1/3 of the female succeed.2) 26 male students succeed.Answer: C

you are right aditya.....

From I, you cannot determine the total number.From II, you still cannot determine the female students.

From I and II, you still cannot determine the number of

females...

Hence, E.

cheers

Back to Top  

TianlongGMAT Tutor

Joined: 16 September 2004United States

Posts: 3843Gender: Male

Posted: 19 August 2005 at 7:31am | IP Logged

131.A guy sent an order of sofas and loveseats, if the total cost was $3800, how many sofas did he buy?

1) One sofa costs $800 and one loveseat costs $300.2) There was more than one sofa in the order.Answer: C

Given answer C is correct.Stat 1 alone - two options (1,10) and (4,2)with Stat 2 - only (4,2)

135. Agree with E. Given answer B is wrong. Two variables but we have only one equation with stat 2. Stat 1 is fluff.

Back to Top  

Sudheer700 Club

Posted: 20 August 2005 at 11:06am | IP Logged

Quote: 104.If d1, d2, d3, d4 are four different integers, whether is 0.d1d2+0.d3d4 greater than 1?

Page 97: Mitul's 2007 GMAT Sep

Joined: 22 March 2005India

Posts: 769Gender: Male

1) The least number is 42) (0.d1d2)(0.d3d4)>0.5

From I, does not give us any idea.From II, yes, we can determine that the sum is greater than 1.For the product to exceed 0.5, the individual numbers should be atleast >0.6

Hence, the result is B.

from 1) if least integer is 4, then  0.d1d2+0.d3d4 should be atleast 0.46 + 0.57 which is greater than 1.

So 1) alone is also sufficient.Isn't it?

Back to Top  

SpidermanGMAT Tutor

Joined: 02 May 2005India

Posts: 945Gender: Not Specified

Posted: 20 August 2005 at 11:36am | IP Logged

Quote: Sudheer Quote: 104.If d1, d2, d3, d4 are four different integers, whether is 0.d1d2+0.d3d4 greater than 1?1) The least number is 42) (0.d1d2)(0.d3d4)>0.5

From I, does not give us any idea.From II, yes, we can determine that the sum is greater than 1.For the product to exceed 0.5, the individual numbers should be atleast >0.6

Hence, the result is B.

from 1) if least integer is 4, then  0.d1d2+0.d3d4 should be atleast 0.46 + 0.57 which is greater than 1.

So 1) alone is also sufficient.Isn't it?

Yes sudheer, you are right.. The answer should be D.

cheers,

Back to Top  

adityakhannaVIP~

Joined: 28 March 2005India

Posts: 10Gender: Male

Posted: 21 August 2005 at 12:04pm | IP Logged

hi, wanted to know if this explanation for q 31 would suffice? i found the earlier one a little confiusing.

31.If x and n are positive integers (or integers?), and when (n+1)(n-1) is divided by 24, the quotient is x and the remainder is r. r=?1) 2 is not the factor of n2) 3 is not the factor of n

Page 98: Mitul's 2007 GMAT Sep

Answer: Cstatement 1:  (24x + r) = n^2 - 1 ; thus, as per the 1st statement since 2 is not a factor of n, 2 has to be a factor of n^2- 1; thus, we know that (24x+r) is a factor of 2; but cant find the value of r

statement 2: similalry; since 3 is not a factor of n, it has to be a facotr of n^2- 1 (tried different numbers and it worked); thus we again know (24x+r) is a facotr of 3; but cant find the value of r

when we combine the statements; we know that (24x+r) is a faoctr of both 2&3, thus is a factor of 6; and since the first part (24x) itself is a multiple of 6; r has to be equal to 0 to satisfy the condition that the full term has to be divisible by 6; and answer is thus c.

 

Back to Top  

billgatesVIP

Joined: 14 August 2005Bermuda

Posts: 717Gender: Not Specified

Posted: 21 August 2005 at 12:53pm | IP Logged

96.The range of the male in a team is 14 inches, and the range of the female in the team is 12 inches, what is the range of the team?1) The shortest of the man is three inches more than the tallest woman.2) The tallest man is 75 inches

range of the team (the tallestest man-the shortest woman)

1)range of the male (Tm-Sm=14);range of woman (Tw-Sw=12)

Sm=Tw+3 ,

hence insuff ,we don t know SM or TW yet

2)Tm=75 so 75-Sm=14       ,therefore Sm =61

but we still can not have the range of the team 

stat 2 is insuff

When we combine it is suff

can someone explain to me how to find A as the right answer

thank u

Back to Top  

archangel88GMAT Tutor

Posted: 21 August 2005 at 1:57pm | IP Logged

Page 99: Mitul's 2007 GMAT Sep

Joined: 14 July 2005United States

Posts: 3148Gender: Not Specified

For (47), if n is an integer and n<-1, then m^n is a fraction and, therefore, not an integer. 

Example:  m=-2, n=-2 => -2^-2 = 1/-2^2 = 1/4

Back to Top  

billgatesVIP

Joined: 14 August 2005Bermuda

Posts: 717Gender: Not Specified

Posted: 21 August 2005 at 4:38pm | IP Logged

131.A guy sent an order of sofas and loveseats, if the total cost was $3800, how many sofas did he buy?1) One sofa costs $800 and one loveseat costs $300.2) There was more than one sofa in the order.

Not sure of the answer.From I alone we can determine the answer, becuase without involving both the terms the sum does not end up to 3800.

My answer : A.

Given Answer: C

it can t be A,because there is more than 1 solution for stat 1

for example :800+(300*10) or (800*4)+(300*2)

Back to Top  

iyer_sriniVIP~

Joined: 10 May 2005United States

Posts: 23Gender: Not Specified

Posted: 22 August 2005 at 5:23am | IP Logged

185.Three of the 15 books are defect. If two books are to be selected from the book, what is the probability that both two books are not defect?

Number of ways you can pick 2 books from 15 = 15C2. Number of ways to pick books without defect = 13C2. Probability = 13C2/15C2.

>>>>>>>>>>>>Should'nt this be 12C2

Back to Top  

iyer_sriniVIP~

Joined: 10 May 2005

Posted: 22 August 2005 at 7:03am | IP Logged

217.Bruce bought only $0.32 stamps and $0.23 stamps. How many $0.23 stamps did he buy? 1) He bought $0.87 worth of stamps. 2) ...

Page 100: Mitul's 2007 GMAT Sep

United StatesPosts: 23Gender: Not Specified

>>>>>>>>>>>> .87 =2*.032 + 1*.23

A. should be sufficient

Back to Top  

adityakhannaVIP~

Joined: 28 March 2005India

Posts: 10Gender: Male

Posted: 22 August 2005 at 10:22pm | IP Logged

hi,

itll be great if someone can just explain this? am not following completely.

64.Six colours (red.black.white.orange.pink.yellow) can be used to decorate. If one or more can be used, how many ways are possible that white is used? Answer: 321+C1,5+C2,5+C3,5+C4,5+C5,5=32

Back to Top  

vcbabuVIP

Joined: 11 September 2004India

Posts: 3078Gender: Not Specified

Posted: 23 August 2005 at 9:48am | IP Logged

Q 64: .Six colours (red.black.white.orange.pink.yellow) can be used to decorate. If one or more can be used, how many ways are possible that white is used? Answer: 321+C1,5+C2,5+C3,5+C4,5+C5,5=32

i think  answer is to be 2^n - 1 = 2^6- 1 ( becose of one or more means at least one ) .

--------------------------

185.Three of the 15 books are defect. If two books are to be selected from the book, what is the probability that both two books are not defect?

Number of ways you can pick 2 books from 15 = 15C2. Number of ways to pick books without defect = 13C2. Probability = 13C2/15C2.

>>>>>>>>>>>>Should'nt this be 12C2

I too think that it shd be 12 C 2

CAn so confirm thse answers .

____________________________________________________________ _________

Back to Top  

Page 101: Mitul's 2007 GMAT Sep

adityakhannaVIP~

Joined: 28 March 2005India

Posts: 10Gender: Male

Posted: 23 August 2005 at 11:10am | IP Logged

80.1/2+1/4+1/8+...+1/512=? A.0<S<1/2 B.1/2<S<1 C.1<S<3/2 D. 3/2<S<2 E. 2<S<5/2

This is geometric progression.a= 1/2, r= 1/2 ,l = 1/512Sum = (1/2 - 1/512)/(1/2) = 255*2/512 = 255/256 < 1.

hi, just wanted to clarify this formula. there might be a slight error?

solved for the q; Sum = 511/512 <1; even though the answer still remains as B.

the numerator says.. a(1 - r^n)..= (a- a*r^n) which is not the same as (a-l)now, a*r^n is not the same as the last number in the sequence, since the last sequence is a*r^n-1; that might be the reason for the difference in answers

Back to Top  

SpidermanGMAT Tutor

Joined: 02 May 2005India

Posts: 945Gender: Not Specified

Posted: 23 August 2005 at 12:03pm | IP Logged

Quote: iyer_srini 217.Bruce bought only $0.32 stamps and $0.23 stamps. How many $0.23 stamps did he buy? 1) He bought $0.87 worth of stamps. 2) ...

>>>>>>>>>>>> .87 =2*.032 + 1*.23

A. should be sufficient

Was not sure of B.. so, did not add any OA..

cheers

Back to Top  

SpidermanGMAT Tutor

Posted: 23 August 2005 at 12:11pm | IP Logged

Hello vcbabu and adityakhanna,

Quote: vcbabu

Q 64: .Six colours (red.black.white.orange.pink.yellow) can be used to decorate. If one or more can be used, how many ways

Page 102: Mitul's 2007 GMAT Sep

Joined: 02 May 2005India

Posts: 945Gender: Not Specified

are possible that white is used? Answer: 321+C1,5+C2,5+C3,5+C4,5+C5,5=32

i think  answer is to be 2^n - 1 = 2^6- 1 ( becose of one or more means at least one ) .

--------------------------

185.Three of the 15 books are defect. If two books are to be selected from the book, what is the probability that both two books are not defect?

Number of ways you can pick 2 books from 15 = 15C2. Number of ways to pick books without defect = 13C2. Probability = 13C2/15C2.

>>>>>>>>>>>>Should'nt this be 12C2

I too think that it shd be 12 C 2

CAn so confirm thse answers .

1. I used logic for the problem.

1 -> with only white5C1 - > white is already taken and the possibility of a total of 2 colors is possible if we can chose one more color from 5 colors.Similarly for the other combinations as well.

  the formula holds good !!

2. It should be 13C2. I am not sure why you decided on 12.If two books are defected, the remaining left are 13. we need to chose from the remaining 13.

[QUOTE=adityakhanna]

80.1/2+1/4+1/8+...+1/512=? A.0<S<1/2 B.1/2<S<1 C.1<S<3/2 D. 3/2<S<2 E. 2<S<5/2

This is geometric progression.a= 1/2, r= 1/2 ,l = 1/512Sum = (1/2 - 1/512)/(1/2) = 255*2/512 = 255/256 < 1.

hi, just wanted to clarify this formula. there might be a slight error?

solved for the q; Sum = 511/512 <1; even though the answer still remains as B.

the numerator says.. a(1 - r^n)..= (a- a*r^n) which

Page 103: Mitul's 2007 GMAT Sep

is not the same as (a-l)now, a*r^n is not the same as the last number in the sequence, since the last sequence is a*r^n-1; that might be the reason for the difference in answers[\QUOTE]

I assumed that the series ends there and hence assumed  l = a*r^n

cheers

Edited by Spiderman on 23 August 2005 at 12:14pm

Back to Top  

iyer_sriniVIP~

Joined: 10 May 2005United States

Posts: 23Gender: Not Specified

Posted: 23 August 2005 at 12:31pm | IP Logged

185.Three of the 15 books are defect. If two books are to be selected from the book, what is the probability that both two books are not defect?

>>> 3 books are defective.

Thanks

 

 

Back to Top  

archangel88GMAT Tutor

Joined: 14 July 2005United States

Posts: 3148Gender: Not Specified

Posted: 25 August 2005 at 1:38am | IP Logged

For 197, shouldn't the formula go something like this:

Range = [9/5(E) + 32] - [9/5(A) + 32]

Back to Top  

SpidermanGMAT Tutor

Posted: 25 August 2005 at 1:49am | IP Logged

Quote: iyer_srini

185.Three of the 15 books are defect. If two books are to be selected from the book, what is the probability that both

Page 104: Mitul's 2007 GMAT Sep

Joined: 02 May 2005India

Posts: 945Gender: Not Specified

two books are not defect?

>>> 3 books are defective.

Thanks

  

Sorry, i overlooked the question...

cheers

Back to Top  

SpidermanGMAT Tutor

Joined: 02 May 2005India

Posts: 945Gender: Not Specified

Posted: 25 August 2005 at 1:53am | IP Logged

Quote: archangel88

For 197, shouldn't the formula go something like this:

Range = [9/5(E) + 32] - [9/5(A) + 32]

Yes. you are right...

cheers

Back to Top  

TianlongGMAT Tutor

Joined: 16 September 2004United States

Posts: 3843Gender: Male

Posted: 25 August 2005 at 6:03am | IP Logged

Hey Spidey ,Don't you think this post is getting unweildy and we should rather restrict discussions to the smaller threads. Let this master thread be only for approved solutions and explanations.

Back to Top  

saurya_s700 Club

Posted: 25 August 2005 at 6:35am | IP Logged

105.n+k=0?

Page 105: Mitul's 2007 GMAT Sep

Joined: 19 September 2004United Kingdom

Posts: 250Gender: Not Specified

1) nk=0 2) n-k=0

From I, we cannot determine the sume. From II, we still cannot determine. From I and II, yes we can determine. (n-k)^2 + 4*nk. Hence, C.

Can someone explain this? Is B not enough? b means n=k. so we get n=k=5 or n=k=-5. in either case sum is not zero and we get No as the answer. So should it be B?

Back to Top  

SpidermanGMAT Tutor

Joined: 02 May 2005India

Posts: 945Gender: Not Specified

Posted: 25 August 2005 at 7:02am | IP Logged

hey all,

Please do not post any questions here.

The respective topics are a new thread in the same forum. The links for the same are provided in the post.

If there is no forum for those set of questions, please start one, and the respective link will be updated..

If needed, PM me. I will always respond to PMs

cheers,

Back to Top  

manugoel007VIP~

Joined: 03 June 2005India

Posts: 18Gender: Not Specified

Posted: 26 August 2005 at 2:24am | IP Logged

Quote: saurya_s 105.n+k=0? 1) nk=0 2) n-k=0

From I, we cannot determine the sume. From II, we still cannot determine. From I and II, yes we can determine. (n-k)^2 + 4*nk. Hence, C.

Can someone explain this? Is B not enough? b means n=k. so we get n=k=5 or n=k=-5. in either case sum is not zero and we get No as the answer. So should it be B?

How about n=k=0....i guess this is self explanatory

 

Back to Top  

TianlongPosted: 26 August 2005 at 11:58pm | IP Logged

Page 106: Mitul's 2007 GMAT Sep

GMAT Tutor

Joined: 16 September 2004United States

Posts: 3843Gender: Male

Z should be 1/24.Answer for this 9/2 or 4.5 times the given amount.

Quote: Spiderman

242. X can transport in 1 hr : 1/16Y in : 1/12 and Z in:1/18total work in 1 r = 1/16 + 1/12 + 1/18In 24 hr = 6/4 + 2/1 + 4/3 = 18 + 24 + 16/12 = 58/12 units

Back to Top  

SpidermanGMAT Tutor

Joined: 02 May 2005India

Posts: 945Gender: Not Specified

Posted: 28 August 2005 at 6:30am | IP Logged

Thanks Tianlong....

cheers

Back to Top  

kg123VIP~

Joined: 20 June 2005India

Posts: 1067Gender: Not Specified

Posted: 28 August 2005 at 8:00am | IP Logged

30.If n and m are positive integers and m=n^3/1800, what is the least value of m?Answer: 151800=(3^2)(2^3)(5^2), n^3=(3^3)*(2^3)(5^3)=15*(3^2)*(2^3)(5^2), so, m=15

Understood : 1800=(3^2)(2^3)(5^2), Did not follow the rest of the steps. Can anyone explain pls. Spiderman, Tianlong : Can u guys chip in.

46.P = sum of positive odd numbers which are < 50Q = sum of positive even numbers which are < 50

Different ways to calculate P,

METHOD I ---- P = 25/2(1 + 49) = 625 ..PROGRESSION....How to arrive at this formula?

51.If 37/13=2+1/(13/x), x=?

Page 107: Mitul's 2007 GMAT Sep

Simplifyin we get, 26+x/13 = 37/13... Hence, x =11

My answer differs : 37/13 = 3/13/x

=37/13 = 3x/13

=37*13 = 39x

=481/39 = x

Pls correct if i am missing something

60.If three people are to be selected from eight people, who include Bob and Nancy, how many ways are possible that Bob, not Nancy is selected?

We have 6 ppl + Bob + Nancy. We drop nancy and have Bob.

We need to select 2 ppl from 6. 6C2 = 15 ( How is this? we need to select 2 ppl from 7 people right, because we dropped only nancy. So 8-1 is 7)Answer: 15 ( Also can anyone explain how 6c2 is 15. Can u detail.)

 

Back to Top  

NC2005VIP~

Joined: 04 July 2005India

Posts: 68Gender: Not Specified

Posted: 01 September 2005 at 5:05am | IP Logged

kg123,

30th Question even i am not able to decipher

46th Question Spiderman used this formula for Arithmetic Progession.        & nbsp;   Sum = n/2 ( First term + Last term)  n is the number of terms

60th Question u r right that we have dropped Nancy but Bob is always present so we have 2 places left for which we have 6 people to choose from

Can anyone pls explain these

30.If n and m are positive integers and m=n^3/1800, what is the least value of m?Answer: 151800=(3^2)(2^3)(5^2), n^3=(3^3)*(2^3)(5^3)=15*(3^2)*(2^3)(5^2), so, m=15

Page 108: Mitul's 2007 GMAT Sep

31.If x and n are positive integers (or integers?), and when (n+1)(n-1) is divided by 24, the quotient is x and the remainder is r. r=?1) 2 is not the factor of n2) 3 is not the factor of n Answer: C

68.If x is a two-digit number less than 50 and x=R5, R=? (R is the tens' digit of x)1) The thousands' digit of x^2 is 12) The hundreds' digit of x^2 is 2Answer: A

69.A cuboid is 25 by 10 by 5, except the bottom, all the side faces are covered with paper. (the bottom maybe 10 by 5?) How much paper will be used?

 

Back to Top  

SpidermanGMAT Tutor

Joined: 02 May 2005India

Posts: 945Gender: Not Specified

Posted: 01 September 2005 at 5:40am | IP Logged

Could NOT add anymore in the top post.. so posted here

251.A delivery company charges $20 for the parcel less than 10 pounds. If the parcel is more than 10 pounds, charge 2 dollars more for each pound exceed 10. Someone had two parcels delivered, how much did he spend?1) The cost to one of the parcels is $2020 The total weight of two parcels is 19.5 pounds.

From I, we cannot tell how much it is less than 10 pounds.. For all the different weihts below 10, will cost 20.From !!, we cannot determine the cost for 0.5 pounds...

hence, E.Answer: E

252.Price of hamburger increased from $1.85 to $2.05, if the price of turkey burger has increased in the same percentage, what is the current cost of turkey burger? The original price of turkey burger is $1.39 £¨number could be wrong£©Answer: 1.39 * [(2.05-1.85)/1.85] +1.39

253.2 teacher and 3 children queue up. There must be one teacher at the beginning and one teacher at the back. 3 children are in the middle. How many ways are possible?

the two teachers at eighter ends, can shuffle themselves in 2 ways.

the remaining children can shuffle in 3! ways.

Hence, 2*3!

Answer: P(3,3)*P(2,2)

254.Xy plane: the name of xy is replaced p, q, a line passed through (0,4) and (4,,0), what is the equation of such line in

Page 109: Mitul's 2007 GMAT Sep

terms of p and q?

The basic equations comes down to x/4 + y/4 = 1

Answer: P+Q=4

255.The value of house increased x % from 1975 to 1980. The value increased y% from 1980 to 1985. Ask how much does the value of the house increased? The house was 500,000 In 1975 (forgot the exact number)1) x + y = a number2) (1+x/100)(1+y/100) = a number

from I, it does not matter, if it adds up to a number.From II, its sufficient to determine the increase in value...

My Answer :B

Given Answer: C

256.A seller bought 100 apples at $0.20 each. He sold 75 of them in a pack of 3, $1 per pack. The rest are sold $0.5 each. What is the average profit for each apple?Answer: [75/3 * 1 + (100-75)*0.5 - 0.2*100] /100

257.How many of 7 consecutive integers can be divided evenly by 6?1) The median is 12.2) The mean is divisible by 6 (old JJ but modified)

From I, it is clear that, from 7 consecutive numbers only a single multiple of 6 exists if its a median.The only way that 2 can exist is when both the ends have a multiple of 6.

From II, The mean turns out to be the 4th element, which is the center element. Hence, same as I.

Hence, D.

Answer: DQ199.Of seven consecutive integers, how many are multiple of 6?1) The median is the multiple of 62) The sum of the numbers is the multiple of 6Answer: D

Same as above.

258.DS: Student major in MIS has to take prerequisites class. 20 of them took account. Some need to take Computer Science. How many of them took both?1) There are 50 students2) All the student took at least one prerequisite class.

From I, we cannot determine the number of students who attended comp classes...From II, still we cannot determine the number since, the number can range from 1 to 50.

hence, E.

Answer: E

259.{number} represents the least integer that is greater than

Page 110: Mitul's 2007 GMAT Sep

the number. If {x/2}= 0, which of the following is the positive value of x?Choices are -2/3, -3. others are all positive.

I would pick -2/3 ,since it is greater than -3 and for -3 it would be -2.

260.A sequence: 2, 4, 6, ...22. Sequence M contains eight numbers, which are elements of the former sequence. What is the standard deviation of M?1) Two sequences have the same average value.2) M does contain 22For standard deviation, we need to know the complete sequence.From I, it cannot be determined.From II, we still do not know the remaining 7.

From I and II, no possibility.

hence, E.

----E

261.Three machines complete a work in 36 hours at the same constant, the same rate, and work for the same time. If add one more machine, how many FEWER hours needed to complete the work?Answer: 9

Already Discussed above

262.x@y = y/x ¨C x/y, ask which of the following must be true? @ represent a functionI. 1/x @ 1/y = bla blaII, and III. Total three equations. I chose the second and the third.

Question Not complete

263.If x is divided by 5, the remainder is 3, divided by 7, the remainder is 4. If y is divided by 7, the remainder is 4. x is greater than y. Which of the following must be the factor of x-y?15, 28, 35

Pluggin and check

for X, it has to be 18, 53.. and so on..

Y has to be 25, 60..

I have understood so far.. But the Q is not clear...

Given Answer: 35

264.Is x>y?1) xy^2>02) xy^3>0

From I, we do not know the comparision between X and Y.

From II, it is possible that x and Y, both are negative.

Page 111: Mitul's 2007 GMAT Sep

from I and II, it is clear that both x and Y are +ve. But still,we do not know the difference.

Answer: E

265.What is the greatest value of the difference between the two close prime numbers from 1 to 30?Answer: 629-23=6

266.If a+b=c+d, where a, b, c, and d are integers. How many of them could be even?Answer: A:0,2,4Odd+odd=odd+odd-->0Even+even=odd+odd-->2Even+odd=even+odd-->2Even+even=even+even-->4

267.If the factorial of 12 is divisible by 2^n, what is the greatest value of n?

12/2 + 12/4 + 12/8 = 6 + 3 + 1 = 10

Given Answer: 101+2+1+3+1+2=10(2,4,6,8,10,12)

268.Both the line's intersections with axis-x and axis-y are 4, which equation represents the line?Answer: x+y=4

Already solved above

269.To complete a certain work, A and B needs 3 and 6 hours, respectively. How many hours will it take for them together to complete half of the work?

Together they take 1/3 + 1/6 = 3/6 = 2 hrs to complete. For Half the work, they take 1 hr.

Answer: 1 hour

270.Is a certain number the multiple of 36?1) It is the multiple of 122) It is the multiple of 20

From I, we cannot determine. eg = 24From II, we still cannot determine.. eg =40

From I and II, we still cannot determine, eg = 60

Answer: E

271.Is x>y?1) x^2>y^22) y-y^2<0

Answer: E

Answered above

272.Someone spent M dollars on some newspapers, magazines, and books. How much did he spend on magazines?1) The cost on books to the cost on newspapers is ...2) The cost on magazines to the cost on newspapers and

Page 112: Mitul's 2007 GMAT Sep

books is ...

From I, we cannot determine the cost of magazine.From II, we can determine the cost of magazine, as we know the total cost and and the ratio.

Answer: B

273.A rectangle is L long and W wide, and has diagonal D. What is the area of the rectangle?1) L+W=Q2) D^2=B

From I, we cannot determine the individual components ,. i.e, L or W.From II, we can only detemine B or D.From I and II, we can determine the area as we know B, and also L now.

Answer: C

274.|x| = |y|, x/y =1? 1) x>0  2) y<0

From I, we know that x is +ve. We do not know abt Y.From II, we do not know abt x.From I and II, we know abt both the quantities...

Answer: C

275.If x is unit digit, y is tenth digit of a certain number, what x and y of  (123,456,789)^2?I. x = 1, II. Y=2 ,III Y=4A) IB) IIC) IIID) I,IIE) I, IIIAnswer: DQ101.What is the tens' digit and the units' digit of 123456789^2?Answer: 21123456789^2=(123456700+89)^2--->89^2

Back to Top  

gmataghuVIP~

Joined: 12 June 2005United Kingdom

Posts: 123Gender: Male

Posted: 04 September 2005 at 2:32am | IP Logged

170.Someone invested a certain amount of money at the 2.5 percent annual interest rate. At the end of the fifth year, the total amount in the account is $11250. How much did he invest?The number maybe inaccurate, pay attention on the thinkingx*(1+2.5%)^5=11250x+12.5%*x+10*2.5%^2x+...(can be neglected)=11250Above all: x*(1+2.5%)^5 approximately equal to x+12.5%*x

Here they have not mentioned whether it is simple interest or compound interest so by default considering it as simple interest the amount will be 10000.

Page 113: Mitul's 2007 GMAT Sep

Criticism welcomed !!

 

Back to Top  

gmataghuVIP~

Joined: 12 June 2005United Kingdom

Posts: 123Gender: Male

Posted: 04 September 2005 at 4:30am | IP Logged

181.P is an integer, is P a prime number?1) P+3 is a prime number2) P^2+3 is a prime number

From I, we can determine that, if P+3 is a prime number, P cannot be a Prime number.Sine, to be a Prime number, the number should be an odd number.From II, P^2 +3 is a prime number, P^2 is an even number, hence, P cannot be prime

Answer: D

------------------------------------------------------------ -------

My answer C because we cannot answer from condition I that whether its prime or not because 2 is also a prime number.

Combining both the conditions we get the answer as 2.

Because 2+3=5 is prime and 4+3=7 is prime. Only 2 satisfies both condition.

Answer C

Open to debate.

Rgds,

Gmataghu

Back to Top  

vcbabuVIP

Joined: 11 September 2004India

Posts: 3078Gender: Not Specified

Posted: 06 September 2005 at 2:44pm | IP Logged

Q60:

60.If three people are to be selected from eight people, who include Bob and Nancy, how many ways are possible that Bob, not Nancy is selected?

We have 6 ppl + Bob + Nancy. We drop nancy and have Bob.

We need to select 2 ppl from 6. 6C2 = 15

Page 114: Mitul's 2007 GMAT Sep

YEAH it is confusing

SO pls explain

Back to Top  

SpidermanGMAT Tutor

Joined: 02 May 2005India

Posts: 945Gender: Not Specified

Posted: 06 September 2005 at 3:25pm | IP Logged

Quote: vcbabu

Q60:

60.If three people are to be selected from eight people, who include Bob and Nancy, how many ways are possible that Bob, not Nancy is selected?

We have 6 ppl + Bob + Nancy. We drop nancy and have Bob.

We need to select 2 ppl from 6. 6C2 = 15

YEAH it is confusing

SO pls explain

We drop Nancy right.... so we have to pick 3 ppl from 7. Are you with me up to here..?

Now we select Bob, hence, we have to pick 2 more ppl from 6. Hence, 6C2 = 15.

Cheers,

Back to Top  

gmataghuVIP~

Joined: 12 June 2005United Kingdom

Posts: 123Gender: Male

Posted: 10 September 2005 at 6:44pm | IP Logged

227.What is the average of 59 consecutive integers?1) The sum of the largest number and the least number is 12) The sum of all is 1

From condition 1 can be true only for even number of cosecutive integers. Consider -2 to 3 there are total six integers similary -28 to 29 are 60 integers not 58 integers not 59( Because there is a 0 inbetween) So we cant get the answer from 1. because it is not possible.

My ans - B

Back to Top  

Page 115: Mitul's 2007 GMAT Sep

gmataghuVIP~

Joined: 12 June 2005United Kingdom

Posts: 123Gender: Male

Posted: 12 September 2005 at 6:00pm | IP Logged

259.{number} represents the least integer that is greater than the number. If {x/2}= 0, which of the following is the positive value of x?Choices are -2/3, -3. others are all positive.

I would pick -2/3 ,since it is greater than -3 and for -3 it would be -2.------------------------------------------------------------ ------------------------------

If I understand this properly, then x should be a number between -1 and 0. And they have asked what is positive value of x , then it should be between 1 and 0.

Correct me if I am wrong.

Back to Top  

NChaudhryVIP~

Joined: 29 July 2005India

Posts: 8Gender: Not Specified

Posted: 13 September 2005 at 12:24pm | IP Logged

In Ques 104 shouldnt the Ans be A Cause its not given whether the nos are positive or negative--

if they are negative then if multiplied they may be greater than 0.5 but if added thy will be less than 1

Back to Top  

INDIGOVIP~

Joined: 30 August 2005United States

Posts: 270Gender: Not Specified

Posted: 17 September 2005 at 5:27pm | IP Logged

SPIDERMAN,

I still don't understand what is happening here... I also read the discussion link..

how do you get n^3=(3^3)*(2^3)(5^3)=15*(3^2)*(2^3)(5^2),

thanks

30.If n and m are positive integers and m=n^3/1800, what is the least value of m?Answer: 151800=(3^2)(2^3)(5^2), n^3=(3^3)*(2^3)(5^3)=15*(3^2)*(2^3)(5^2), so, m=15

Discussions MJJ30-48

Back to Top  

Page 116: Mitul's 2007 GMAT Sep

rnk_m700 Club

Joined: 21 May 2005United States

Posts: 663Gender: Not Specified

Posted: 21 September 2005 at 8:03pm | IP Logged

Quote: INDIGO

SPIDERMAN,

I still don't understand what is happening here... I also read the discussion link..

how do you get n^3=(3^3)*(2^3)(5^3)=15*(3^2)*(2^3)(5^2),

thanks

30.If n and m are positive integers and m=n^3/1800, what is the least value of m?Answer: 151800=(3^2)(2^3)(5^2), n^3=(3^3)*(2^3)(5^3)=15*(3^2)*(2^3)(5^2), so, m=15

Discussions MJJ30-48

explanation

Back to Top  

INDIGOVIP~

Joined: 30 August 2005United States

Posts: 270Gender: Not Specified

Posted: 21 September 2005 at 10:21pm | IP Logged

153.DS: X<5?1). |x-5|>02). x^2 + x <5

From I, we cannot determine if x<5. If X is 4 or 6, we still get an answer of 1.from II, it solves as x(x+1) <5, in both ways, x is eighter 0 or less than 4. Hence, B. 

FOR STATMENT 2.HOW DO WE KNOW THAT "X IS EITHER 0 OR LESS THAN 4"???

Back to Top  

kgKid700 Club

Posted: 25 September 2005 at 9:02pm | IP Logged

Quote: gmataghu

181.P is an integer, is P a prime number?1) P+3 is a prime number2) P^2+3 is a prime number

From I, we can determine that, if P+3 is a prime number, P

Page 117: Mitul's 2007 GMAT Sep

Joined: 15 August 2005United States

Posts: 87Gender: Not Specified

cannot be a Prime number.Sine, to be a Prime number, the number should be an odd number.From II, P^2 +3 is a prime number, P^2 is an even number, hence, P cannot be prime

Answer: D

------------------------------------------------------------ -------

My answer C because we cannot answer from condition I that whether its prime or not because 2 is also a prime number.

Combining both the conditions we get the answer as 2.

Because 2+3=5 is prime and 4+3=7 is prime. Only 2 satisfies both condition.

Answer C

Open to debate.

Rgds,

Gmataghu

How about P to be 4. It satisfies both the conditions. Infact, the answer should be E.

 

Back to Top  

antmavelVIP~

Joined: 08 September 2004China

Posts: 245Gender: Male

Posted: 27 September 2005 at 10:34pm | IP Logged

141.What is the range of the numbers 2, 3, 4, 6, 7, 9, 10, 12, 20, and x?1) The mean is 92) x is the median

Answer :D

If the mean is 9, we can determine the number x.From II, if x is the median, we can determine x, from the given series.

Answer is correct but explanation is wrong.

Statement 1 -> agree with you

Statement 2 -> we can not determine X, it could be 7,8 or 9. However we know that it's neither the smallest nor the biggest so we can calculate the range : 20-2 = 18

Page 118: Mitul's 2007 GMAT Sep

Back to Top  

gears46VIP~

Joined: 02 December 2004United States

Posts: 401Gender: Not Specified

Posted: 08 October 2005 at 3:01pm | IP Logged

88.Of a group of people donate for a organization, 1/4 donate less than $200, 2/3 donate from $200 to $1000, and others donate more than $1000. If the average value of the donation more than $200 is 360, what is the average value of the donations more than $1000?1) The average value of the donations less than $200 is 1802) The average value of the donations from $200 to $1000 is $540

From I alone, we cannot determine the avg, since, we do not have the avg from 200$ to  1000$.

From II, yes, we can determine.

We have the information that , the avg of donations more than $200 is 360.

Let x, be the number of ppl who donated money.Then the number of ppl who donated b/w 200 and 1000 is 2/3 x.and those beyond 1000 is 1 - (1/4+2/3). = 1/12x.

hence, we can calculate, (540*2/3x + A *1/4X) /2/3x + 1/4X = 360

Answer: B

Spiderman,

I believe that the answer would be C.  Cause in the formula that you give you have two variables (A and x).  Where you have "A" standing for the average of the donations less than $200. If you plug in that value (which is 180) into your equation, then you can solve the total number of people (which is "x").

However, when I did what was stated above, I get a fractional number for the total number of people. I believe that either the problem has a number or twop that is incorect, or if those were the intended numbers, the answer is "E" (cause you can't have a fractional part of a person. 

Can you please expand and solve  your equation for #88, cause I really can't see how the answer would be "B".

Thanks,

 

Gears46

Quote: clueless26

@ Orange&Orange & Kristi

Page 119: Mitul's 2007 GMAT Sep

11 & 5 have a set pattern when raised to a particular power... i mean for 5: for every odd power <= 3 the last 3 digits would be 125

for every even power < 2 the last 3 digits would be 625.

for 11..the tens digit would depend on the power raised...

if n = 2,12 etc the tens digit is 2,

n =3,13 tens digit is 3 so on....

not sure if this would help but 76 & 25 are the only 2 digit numbers that when raised to any power end with the same number.. ie...76^2 ,76^3,76^100 will have 76 as the last 2 digits. Same with 25

26 raised to any power has the last 2 digits as 76

25 raised to any power has the last 3 digits as 625

Very nice tip. Thanks.

One minor mod, I believe clueless meant:

11 & 5 have a set pattern when raised to a particular power... i mean for 5: for every odd power >= 3 the last 3 digits would be 125

for every even power > 2 the last 3 digits would be 625.

82. The surface of a Clock (in a dart game) is evenly divided to eight parts numbered with numbers 1 to 8. If we throw arrows three times, how many ways are possible that the total score is 16?42, 56, 64…

Back to Top  

anil_bitsVIP~

Joined: 24 August 2006India

Posts: 82Gender: Male

Posted: 12 September 2006 at 6:54pm | IP Logged

Take x=1 y,z can be (7,8) => 2 combinations x=2 => 3 similar as above 6,8 7,7 8,6 x=3 => 4 x=4 => 5 x=5 => 6 x=6 => 7 x=7 => 8 x=8 => 7 Total 42

Back to Top  

Page 120: Mitul's 2007 GMAT Sep

eclipse00700 Club

Joined: 05 June 2006Korea, South

Posts: 282Gender: Male

Posted: 12 September 2006 at 9:16pm | IP Logged

Yes.. It is 42.

Back to Top  

SerafimVIP~

Joined: 24 March 2006Ukraine

Posts: 428Gender: Male

Posted: 12 September 2006 at 9:30pm | IP Logged

Anil_bits - perfect solution.

Back to Top  

kpinvictusVIP~

Joined: 07 September 2004Senegal

Posts: 189Gender: Male

Posted: 12 September 2006 at 9:51pm | IP Logged

agree

__________________Time is the essence and sky the limit!

Back to Top  

VSGMAT700 Club

Posted: 12 September 2006 at 11:38pm | IP Logged

When throwing a dart, there is a possibility of missing all together. I guess we should also consider the case 0 8 8. According to me, favourable cases are as follows:

Page 121: Mitul's 2007 GMAT Sep

Joined: 24 May 2006India

Posts: 109Gender: Male

8 8 0 ==> 3 combinations8 7 1 ==> 6 combinations8 6 2 ==> 6 combinations8 5 3 ==> 6 combinations8 4 4 ==> 3 combinations7 7 2 ==> 3 combinations7 6 3 ==> 6 combinations7 5 4 ==> 6 combinations6 6 4 ==> 3 combinations6 5 5 ==> 3 combinations

Total no of favourable combinations :  45 (42 + 3 [for 8 8 0])

Back to Top  

tmddpVIP

Joined: 09 January 2006India

Posts: 1651Gender: Not Specified

Posted: 13 September 2006 at 12:26am | IP Logged

Good explanation VSGMAT. But I think, the 8 8 0 condition might not be needed. Depends on the options though

__________________Success is the sole earthly judge of right and wrong

Back to Top  

anupam384700 Club

Joined: 23 August 2006India

Posts: 99Gender: Male

Posted: 13 September 2006 at 12:37am | IP Logged

where i m missing

1 7 8 -->6 cases  ( 3! )

2 8 6 -->6 cases

3 8 5 -->6 cases

4 8 4 -->3 cases  

total 21 cases??????????????????

__________________I will not go quietly into the night , I will not vanish without a fight , I am going to live on , I am going to survive..

Page 122: Mitul's 2007 GMAT Sep

Back to Top  

DarshanVIP

Joined: 08 September 2005India

Posts: 665Gender: Male

Posted: 13 September 2006 at 12:45am | IP Logged

Any short cut method ??Creation of this possibility table takes time.

Back to Top  

ashok_nitw700Club

Joined: 27 July 2006India

Posts: 300Gender: Male

Posted: 13 September 2006 at 1:06am | IP Logged

42 is the answer (a throw gets min 1, so 8 8 0 combination is not possible)

anupam u r missing some combination .. to avoid such mistakes take numbers systematically like below

8 7 1 - 3! --> 6 8 6 2---------> 6 8 5 3----------> 6 8 4 4---3!/2 ---> 3 7 7 2----------->3 7 6 3----------->6 7 5 4------------>6 6 6 4------------->3 6 5 5-------------->3 any further combinations r duplicate sets.

5* 6 + 4 * 3 = 42.

there is no need to list like this every time just count the combinations which have all different numbers,which have 2 same and remaining different.. correspondingly use arrangements formula to get the answer.

Edited by ashok_nitw on 13 September 2006 at 1:30am

__________________The only true wisdom is in knowing you know nothing

Back to Top  

ashok_nitw700Club

Posted: 13 September 2006 at 1:07am | IP Logged

.

Edited by ashok_nitw on 13 September 2006 at 1:29am

Page 123: Mitul's 2007 GMAT Sep

Joined: 27 July 2006India

Posts: 300Gender: Male

__________________The only true wisdom is in knowing you know nothing

Back to Top  

alfa_beta02VIP~

Joined: 12 July 2006United States

Posts: 1188Gender: Not Specified

Posted: 13 September 2006 at 3:16am | IP Logged

I was able to solve the question but it took me around 5-6 mins, may be more, to get the answer. Is there any shorter approach to solve this problem?

Back to Top  

Orange&OrangeVIP

Joined: 07 September 2006Antarctica

Posts: 274Gender: Not Specified

Posted: 18 September 2006 at 6:37pm | IP Logged

Quote: alfa_beta02 I was able to solve the question but it took me around 5-6 mins, may be more, to get the answer. Is there any shorter approach to solve this problem?

I have the same question as alfa did - does anyone know any shortcuts for such questions?

__________________O&O

Back to Top  

hector81700 Club

Joined: 25 August 2006India

Posts: 53Gender: Not Specified

Posted: 25 September 2006 at 7:15pm | IP Logged

i dont know if it'll work eveytime.. worked in this ques.. since the sum is 16.. the possible combination of three nos can be done in three ways or 6 ways.. for eg.. 1, 7,8 can be form a sum in six ways.. also 2,7,7 can also form the sum in three ways.. that means.. all possible combination can be done in atleast three ways.. noting the options there was only one nos which was divisble by three.. thats 42.. thats how i chose..

this aproach will be wrong when there r other options which will be divisible by 3..

Page 124: Mitul's 2007 GMAT Sep

not a gauranteed approach.. but will work in cases like this..

wats say..

Edited by hector81 on 25 September 2006 at 7:18pm

Back to Top  

JadoreVIP

Joined: 03 August 2006Russia

Posts: 1366Gender: Not Specified

Posted: 25 September 2006 at 8:52pm | IP Logged

42

__________________A diamond is a piece of coal that stuck to the job.

Back to Top  

alfa_beta02VIP~

Joined: 12 July 2006United States

Posts: 1188Gender: Not Specified

Posted: 25 September 2006 at 8:58pm | IP Logged

@Jadore,Can you share the approach you used to solve the problem? Am looking for the best approach to solve the problem.

Thanks!